#22 Rosh Review

Pataasin ang iyong marka sa homework at exams ngayon gamit ang Quizwiz!

Question: What kind of prognostic indicator is the presence of B symptoms in Hodgkin lymphoma?

Answer: Poor outcome.

Question: What is the name of the obligate human parasite that causes scabies?

Answer: Sarcoptes scabiei. Rapid Review Scabies Sarcoptes scabiei Pruritic rash worse at night Linear burrows Interdigital spaces of hands/feet, penis, breasts Permethrin Wash clothing, stuffed animals, bedding Treat household contacts.

Question: What complication can occur secondary to prolonged application of lindane?

Answer: Seizures. Rapid Review Pediculosis Head lice, body lice, pubic lice Pruritis Nits: white dots on base of hair shafts 1st line rx: permethrin Lindane: seizures, aplastic anemia

Question: What is a classic ECG finding of hypercalcemia?

Answer: Shortened QT interval.

Question: What is the most common cause of hyperkalemia found on blood work?

Answer: The most common cause is spurious elevation from extravascular hemolysis. Rapid Review Hyperkalemia Patient with a history of renal failure, DKA, rhabdomyolysis, tumor lysis Complaining of lethargy, weakness, paralysis PE will show bradycardia, hypotension, cardiac dysrhythmia ECG will show peaked T waves, prolonged PR, wide QRS Treatment is calcium gluconate, insulin, albuterol, kayexalate, bicarbonate

Question: What is the INR range in a patient taking warfarin for atrial fibrillation?

Answer: 2-3.

Question: What routine lab work should be considered before prescribing oral antifungal medications?

Answer: Liver function tests (LFTs).

Question: What is the best predictor of outcome in submersion injuries?

Answer: Submersion duration.

Question: What lab test is most likely to be abnormal in patients with spinal epidural abscess?

Answer: The erythrocyte sedimentation rate (ESR) is often elevated despite a frequently normal WBC count. Rapid Review Spinal Cord Compression Back pain Urine retention/inability to void Decreased rectal tone Saddle anesthesia Lower extremity weakness

During an exam, passive flexion of a patient's neck causes a reflexive flexion of his hips and knees. What is this finding called? Brudzinski sign Griesinger's sign Kernig's sign Levine's sign

Correct Answer ( A ) Explanation: This finding describes Brudzinski sign, which is used as an indicator of meningeal irritation such as in patients with acute meningitis. The test is performed by applying flexion to the patient's neck. A positive test is the observance of a reflexive flexion of the hips and knees. Care should be taken regarding a positive Brudzinski sign, as this does not always indicate acute meningitis. In addition to meningitis, it can also be observed in patients with subarachnoid hemorrhage or encephalitis. All conditions that cause meningeal irritation. Griesinger's sign (B) is swelling of the posterior auricular area and may be seen with certain types of sinus thrombosis. Kernig's sign (C) is usually assessed along with Brudzinski. It is performed by flexing the hip and knee to 90° and then attempting to extend the knee. This will cause significant pain in a patient with meningeal irritation. Levine's sign (D) is described as leaning forward on a closed fist in the chest area and is sometimes seen in patients with acute coronary syndrome.

A 33-year-old woman is being evaluated in clinic for pain in her left lower leg. She underwent an arthroscopic meniscus repair of the left knee three months ago. She denies any recent trauma. Patient reports sudden onset of pain and burning sensation in her leg. It does not worsen with physical activity or elevation. She denies fever or chills. On physical exam her left leg is extremely erythematous, warm, swollen and tender to the lightest touch. Lower extremity pulses are 2+ bilaterally. Initial lab work reveals a normal complete blood count, erythrocyte sedimentation rate, and C-reactive protein. What is the most likely diagnosis? Cellulitis Complex regional pain syndrome Erysipelas Peripheral vascular disease

Correct Answer ( B ) Explanation: Complex regional pain syndrome is a form of chronic pain in a body region, but it most commonly affects the extremities. It is characterized by pain, swelling, skin changes, vasomotor instability, limited range of motion, and patchy bone demineralization. It frequently occurs after a soft tissue injury, surgery, or vascular event such as a myocardial infarction or stroke. The pathophysiology of complex regional pain syndrome is incompletely understood and most of the time there is no definable nerve lesion. Symptoms of complex regional pain syndrome include pain that is described as burning, tingling, throbbing or aching. The pain can be quite severe and out of proportion to physical exam. Many patients experience allodynia, which is pain from a stimulus that does not usually provoke pain. Other symptoms include sensitivity to cold or touch, swelling, skin changes such as thickened, rough skin, or muscle atrophy. A diagnosis of complex regional pain syndrome requires the presence of regional pain and sensory changes following a noxious event. It is usually made clinically as there is no laboratory or radiologic testing that alone is diagnostic. Both cellulitis (A) and erysipelas (C) present with erythema, warmth, edema and pain. However, these are caused by bacterial infections, most commonly S. aureus and beta-hemolytic streptococci. Cellulitis involves the deeper dermis while erysipelas affects more superficial layers. Erysipelas is characterized by erythema that is raised above the level of surrounding skin with sharply demarcated borders. Cellulitis is more diffuse, without clear borders and is not raised. Both of these conditions are associated with a systemic response such as fevers and chills, usually with a white count and elevated inflammatory markers. Erythrocyte sedimentation rate, C-reactive protein and the peripheral blood white blood cell count are unremarkable in complex regional pain syndrome. Peripheral vascular disease (D) is the result of compromised blood supply to the extremities caused by atherosclerosis in the peripheral arteries. This syndrome is characterized by the 5 P's of pain, pallor, pulselessness, paresthesia, and poikilothermia (cold). This results in cold, pale, extremities and claudication. Claudication is pain or cramping that is worse with activity such as walking.

A 35-year-old man presents with right eye pain after drilling at his construction site five hours earlier. You note a metallic foreign body at the 3 o'clock position on the cornea and remove it using a 27-gauge needle. A rust ring is visible on the cornea. When is the most appropriate time to attempt rust ring removal? 1 hour after development of the rust ring 24 to 48 hours after development of the rust ring 4 to 6 hours after development of the rust ring 5 to 7 days after development of the rust ring

Correct Answer ( B ) Explanation: Metallic foreign bodies embedded in the cornea develop rust rings due to oxidation of the iron. Rust rings can develop within hours of metallic foreign body contact with the cornea. Over 24-48 hours, the rust ring will oxidize and kill surrounding epithelium. At this time, the rust ring will be soft and easily removed in one solid plug. The surrounding epithelium may not be easily removed (A and C) until 24 hours after the rust ring develops. After 5-7 days (D), the rust ring and surrounding epithelium is at risk for infection and scarring. Alternatively, the rust ring may be removed at the same time as the foreign body removal, either with an eye spud or with a rotating bur if the emergency physician is comfortable with this procedure.

Question: How long should an individual undergo treatment for an isolated pulmonary embolism with no other risk factors?

Answer: 3 months. Rapid Review Pulmonary Embolism 95% arise from deep leg veins Sudden onset of symptoms in 50% SOB, CP, tachypnea ECG: sinus tachycardia, nonspecific ST-T changes, right heart strain, S1Q3T3 (classic finding) CXR: nonspecific abnormalities, Hampton's hump (pleural-based wedge infarct), Westermark's sign (vascular cut-off sign) V/Q scan: usually nondiagnostic Low clinical suspicion: negative D-dimer excludes PE Dx of choice: CTPA Treatment: Anticoagulation Thrombolytics (if massive and HD unstable or submassive with shock, respiratory failure or evidence of moderate to severe RV strain) Embolectomy (last resort)

Question: What is the dosing of iron supplementation in a one-year-old child with iron deficiency anemia?

Answer: 6 mg/kg/day divided into three doses.

Question: In diagnosing peripheral artery disease, what is considered an abnormal ankle-brachial index?

Answer: < 0.9. Rapid Review Complex Regional Pain Syndrome Patient will have a history of previous extremity injury or fracture Complaining of light touch causing extreme pain and allodynia (pain felt from a nonpainful stimulus, such as clothes or bed sheets on the skin) Treatment is NSAIDs, gabapentin, sympathectomy

Question: What is urine specific gravity?

Answer: A measure of urine solute concentration. High values indicate a concentrated urine, Low values indicate a dilute urine. Rapid Review Diabetes Insipidus (DI) ADH deficiency → polyuria + inability to concentrate urine ↑ Plasma osmolality + dec urine osmolality Central DI: ↓ ADH production Water deprivation test: >50% ↑ in urine osmolality Rx: intranasal DDAVP Nephrogenic DI: Renal unresponsiveness to ADH Water deprivation test: no change in urine osmolality Rx: HCTZ, amiloride, indomethacin

Question: What agent is recommended for patients with contraindications to metformin?

Answer: A shorter acting sulfonylurea such as glipizide.

Question: What is the most common form of lung cancer?

Answer: Adenocarcinoma.

Question: What type of hypersensitivity reaction is allergic contact dermatitis?

Answer: Allergic contact dermatitis is a form of delayed hypersensitivity (type IV) reaction mediated by lymphocytes sensitized by the contact of the allergen with the skin. Rapid Review Contact Dermatitis Patient will complain of a rash PE will show erythematous, scaly plaques, vesicles, and bullae Diagnosis is made by patch test Most commonly caused by nickel, poison ivy, soaps, and clothing Comments: Cell-mediated reaction type IV

Question: What structures of the knee make up the unhappy triad?

Answer: Anterior cruciate ligament (ACL), medial collateral ligament (MCL), and medial meniscus. Rapid Review Anterior Cruciate Ligament (ACL) Tear Patient with a history of quickly stop moving and change direction while running Complaining of pop and swelling Diagnosis is made by Lachman's Test (most sensitive) and Anterior Drawer Test

Question: What are the risk factors for developing torsade de pointes?

Answer: Bradycardia, hypokalemia, hypomagnesemia, congenital long QT syndrome, and concomitant use of other drugs that cause QT prolongation. Rapid Review Clozapine: Agranulocytosis

Question: What are the main actions of diltiazem?

Answer: Calcium channel blockade into myocardial cells resulting in decreased contraction, AV nodal conduction, and, to a lesser degree, vasodilation. Rapid Review Atrial Fibrillation Alcohol Irregularly irregular No P waves Narrow QRS unless conduction block or accessory pathway Unstable: cardioversion Stable: Rate control with CCBs, ßBs <48 hours duration: cardiovert to sinus rhythm >48 hours duration: anticoagulate, echo to r/o thrombus, then cardioversion Beyond The Boards HAS-BLED is a scoring system developed to assess one-year risk of major bleeding in patients with atrial fibrillation. It was developed in 2010 with data from 3,978 patients in the Euro Heart Survey. A calculated HAS-BLED score is between 0 and 9 and based on eight parameters with a weighted value of 0-2. The HAS-BLED mnemonic stands for: Hypertension, Abnormal renal and liver function, Stroke, Bleeding, Labile INRs, Elderly, Drugs or alcohol.

Question: What is the most commonly isolated pathogen in patients with Guillain-Barré syndrome?

Answer: Campylobacter jejuni.

Question: What group has the highest suicide rate in the United States?

Answer: Caucasian men, aged 85 years and older.

Question: What is the most common type of gallstone?

Answer: Cholesterol gallstone. Rapid Review Cholecystitis Patient will be complaining of colicky, steadily increasing RUQ or epigastric pain after eating fatty foods PE will show Murphy's sign, Boas' sign Diagnosis is made by: Initial: US, Gold standard: HIDA Most commonly caused by obstruction by a gallstone Treatment is cholecystectomy

Question: In which chromosome is the excess CAG repeats seen in patients with Huntington's disease?

Answer: Chromosome 4. Rapid Review Huntington's Disease Autosomal dominant, CAG trinucleotide repeats Age 35-40 Atrophy of caudate nucleus, putamen, globus pallidus Chorea + dementia + labile mood

Question: What is a major complication of infant respiratory distress syndrome?

Answer: Chronic pulmonary disease, namely bronchopulmonary dysplasia, may result from the oxygen and ventilation actually used in its treatment. Rapid Review Infant (Neonatal) Respiratory Distress Syndrome ↓ Fetal surfactant Risk factors: prematurity (most common), maternal diabetes, c-section Respiratory difficulty within a few hours after birth CXR: ground glass appearance Rx: O2, intubation, CPAP, surfactant Concern for premature birth: prenatal glucocorticoids O2 complications: blindness, bronchopulmonary dysplasia

Question: Which cardiac disorder is similar to restrictive cardiomyopathy?

Answer: Constrictive pericarditis, which is caused by viral or TB infection, uremia, radiation and post-cardiac-surgery fibrosis. Rapid Review Restrictive Cardiomyopathy Least common cardiomyopathy Most common cause: amyloidosis Impaired diastolic filling + preserved systolic function Right-sided heart failure symptoms ECG: low voltage, nonspecific changes

Question: What is a complication of corneal abrasion?

Answer: Corneal ulcer with scarring. Rapid Review Corneal Abrasion Patient will be complaining of pain, photophobia, tearing, or foreign body sensation Diagnosis is made by fluorescein stain Treatment: Most are self-limiting, antipseudomonal for contact lens wearers

Question: For which tick-borne illness is doxycycline the recommended treatment regardless of age?

Answer: Ehrlichiosis.

Question: How can the patient be positioned to improve preoxygenation?

Answer: Elevate the head of the bed.

Question: What are the most common species of dermatophytes?

Answer: Epidermophyton, trichophyton, and microsporum.

Question: What is the mode of transmission for Hepatitis A?

Answer: Fecal-oral either through sexual contact or contaminated food sources. Rapid Review Acute Hepatitis HAV: fecal-oral, shellfish, alone (no carrier), asymptomatic, acute HBV: HBsAg: active infection Anti-HBs: recovered or immunized Anti-HBc IgM: early marker of infection, positive in window period Anti-HBc IgG: best marker for prior HBV HBeAg: high infectivity Anti-HBeAb: low infectivity HCV: IVDA, chronic, cirrhosis, carcinoma, carrier HDV: dependent on HBV coinfection HEV: fecal-oral (enteric) high mortality rate among pregnant (expectant) patients, epidemics, HAV and HEV are fecal-oral: "The vowels hit your bowels" Autoimmune hepatitis: young females Alcoholic hepatitis: moderate transaminase elevation, AST>ALT Supportive rx

Question: What is the MRI finding which confirms a diagnosis of hip avascular necrosis?

Answer: Femoral head hypointensity (T1) and double-line signs (T2). Rapid Review Avascular Necrosis Causes: corticosteroids (most common), alcohol Most common site: femoral head MRI

Question: What is the classic triad of renal cell carcinoma?

Answer: Flank pain, hematuria and palpable flank mass. Rapid Review Bladder Cancer Patient will be older With a history of smoking Complaining of painless hematuria Diagnosis is made by cystoscopy Most common type is transitional cell carcinoma

Question: What are the three types of botulism infection?

Answer: Foodborne botulism, wound botulism and infant botulism. Rapid Review Botulism Patient will be an infant With a history of eating honey Complaining of feeble cry, constipation PE will show symmetric descending paralysis ("floppy baby") Most commonly caused by Clostridium botulinum Treatment is IV botulism Ig

Question: To reduce the risk of secondary cases, chemoprophylaxis with oral rifampin is necessary for household contacts and anyone who has had direct exposure to an index case's oral secretions up to how many days before the index case's onset of illness?

Answer: For 7 days. Rapid Review Meningococcemia Military recruits, students Fever, HA, arthralgias, rash Petechiae, skin lesions with gray necrotic centers Waterhouse-Friedrichson syndrome: bilateral adrenal hemorrhage + meningococcemia Ceftriaxone and vancomycin

Question: What does the fourth generation combination HIV-1/2 immunoassay detect?

Answer: HIV-1 antibodies, HIV-2 antibodies and HIV-1 p24 antigen.

Question: What cerebrospinal fluid (CSF) findings are seen in bacterial meningitis?

Answer: High protein, low glucose, presence of polymorphonuclear cells.

Question: Which virus is responsible for Roseola (exanthema subitum)?

Answer: Human herpesvirus 6 (HHV-6). Beyond The Boards In 2004, rubella was officially declared eliminated from the United States and from the Americas in 2015.

Question: Can an MRI be used to identify intraocular foreign bodies?

Answer: If the foreign body is metallic, an MRI is contraindicated because it can dislodge the foreign body and damage surrounding tissue. Rapid Review Corneal Abrasions and Foreign Bodies Metallic foreign bodies: rust rings Slit lamp Fluorescein stain: epithelial injury Most abrasions are self-limiting

Question: In what risk patients is a D-Dimer appropriate in the work up of pulmonary embolism?

Answer: In low risk patients in whom a negative test effectively rules out pulmonary embolism.

Question: What is the drug of choice to close a patent ductus arteriosus in premature infants?

Answer: Indomethacin. Rapid Review Patent Ductus Arteriosus Patient will be complaining of failure to thrive, poor feeding, tachycardia, and tachypnea PE will show continuous, rough, "machinery-like" murmur, heard best in the first interspaces of the LSB Diagnosis is made by echo Treatment is indomethacin

Question: What is the most common precipitating event for hyperglycemic hyperosmolar state?

Answer: Infectious etiology is the most common precipitant for hyperglycemic hyperosmolar state Rapid Review Hyperosmolar Hyperglycemic State Hyperglycemia → dehydration w/o acidosis Type 2 DM AMS Glc > 600 Negative ketones Rx: IVF

Question: What condition should be considered in individuals diagnosed with a right sided varicocele?

Answer: Inferior vena cava thrombosis. Rapid Review Hydrocele Most common cause of scrotal enlargement in children Fluid around the testicle → painless testicular swelling Right > left Transilluminates Indirect inguinal hernia Ultrasound

Question: What is the clinical difference between internal and external hemorrhoids?

Answer: Internal hemorrhoids cause rectal bleeding however they are not painful.

Question: What is a common complication of Meckel's diverticulum?

Answer: Intestinal obstruction caused by intussusceptions or volvulus. Rapid Review Meckel's Diverticulum: Incomplete obliteration of vitelline duct Rule of 2's: 2 years old, 2 feet from the ileocecal valve, 2 inches long, 2 percent of the population Painless bright red GI bleeding Diagnosis with Meckel's scan

Question: What is the reaction that can occur during the treatment of syphilis that causes fever, chills, rash, and myalgias?

Answer: Jarisch-Herxheimer reaction. Rapid Review Primary Syphilis Patient will have been sexually active 2 - 3 weeks ago Complaining of bump on his penis PE will show painless, "Punched out" lesion Diagnosis is made by darkfield microscopy Most commonly caused by Treponema pallidum Treatment is single IM injection of benzathine penicillin Comments: syphiLIS is painLESS ulcer

Question: What are the most common methods of successfully completed suicides and suicide attempts?

Answer: Most successfully completed suicides involve firearms, and most suicide attempts involve ingestions. Rapid Review Suicide Protective factors: marriage, pregnancy RFs (SAD PERSONS): Sex (male), Age (teenager or elderly), Depression, Previous attempt, Ethanol/drug use, Rational thinking loss, Sickness (medical illness, 3 or more prescription medications), Organized plan, No Social support, Stated future attempt Most completed suicides involve firearms Most attempted suicides involve ingestions (MC: antidepressants) All overdose patients: obtain acetaminophen level

Question: In necrotizing (malignant) otitis externa, does cranial nerve involvement affect mortality?

Answer: No. Rapid Review Otitis Externa Patient with a history of swimming or moisture exposure Complaining of malodorous discharge and pruritus PE will show pain with palpation of tragus/pinna Most commonly caused by Pseudomonas aeruginosa Treatment is topical antimicrobials with or without steroids Comments: Necrotizing otitis externa - a complication seen in diabetics/immunocompromised

Question: Name three complications of toxic megacolon?

Answer: Perforation, sepsis and shock. Rapid Review Hirschsprung Disease Congenital megacolon Lack of ganglion cells in colon → ↑ muscle tone, aperistalsis Boys, Down Syndrome Delayed meconium passage Toxic megacolon Barium enema: cone-shaped transitional zone

Question: If there is no response to gabapentin, what other pain medications could be tried?

Answer: Pregabalin, duloxetine and carbamazepine. Rapid Review Diabetic Neuropathy Polyneuropathy Numbness, tingling "Stocking-glove" distribution Rx: TCAs, gabapentin

Question: In patients with appendicitis, what physical sign occurs when right lower quadrant pain is elicited by passive right hip extension?

Answer: Psoas sign. Rapid Review Appendicitis: Patient will be complaining of fever, pain that began periumbilical then moved to RLQ, nausea, and anorexia PE will show Psoas sign (RLQ pain on extension of right hip), Obturator sign (RLQ pain on internal rotation of flexed right hip), Rovsing sign (right lower quadrant pain when the left lower quadrant is palpated) Diagnosis is made by ultrasound, CT Most commonly caused by fecalith Treatment is surgery

Question: Name some causes of infectious cataract?

Answer: Rubella, varicella, toxoplasmosis and cystercercosis. Beyond The Boards Statins and Cataracts: In preclinical toxicity testing, dogs developed cataracts when given doses of statins much higher than human doses. While most large case-control and cohort studies, as well as a small randomized trial, have not found an increased risk of cataract, large cohort studies from England and Wales and from the United States military health system have found that statin use was associated with an increased risk of cataract. In a subset analysis, one cohort study reported an association between statins and a decreased risk of one type of cataract (nuclear cataract). An analysis of safety results from a randomized trial also found an apparent protective effect of lipid lowering therapy with simvastatin plus ezetimibe; however, there were very few cataract events, the types of cataracts were not reported, the result was of borderline statistical significance, and reporting bias is a concern with these sorts of results from safety data. Additional study is clearly needed before it can be concluded that statins actually have any such protective effect. Rosenson, R. Statins: Actions, side effects, and administration. In: UpToDate, Post TW (Ed), UpToDate, Waltham, MA. (Accessed on January 9, 2017.)

Question: What critical diagnoses should be considered in patients with syncope and back pain?

Answer: Ruptured abdominal aortic aneurysm, aortic dissection, pulmonary embolism and ruptured gastric/duodenal ulcer. Rapid Review Back Pain Night pain, weight loss: malignancy Back pain + fever + neurological deficits: epidural abscess Acute bony tenderness: fracture Young, morning stiffness: seronegative spondyloarthropathy Urinary retention: cauda equina syndrome Pain with extension, relief with flexion: spinal stenosis Image if red flags present

Question: Where are the most common sites for Crohn's disease to occur?

Answer: Small bowel 50%, colon 30%, and ileocolonic 20%. Rapid Review Crohn's Disease Involvement: mouth to anus but usually distal ileum, colon, spares rectum Transmural inflammation Bloody diarrhea uncommon Exam: aphthous ulcers, anal fissures, perirectal abscesses, anorectal fistulas Colonoscopy: skip lesions, cobblestone mucosa ASCA positive, p-ANCA negative No surgery Ileitis rx: mesalamine Ileocolitis/colitis: mesalamine or sulfasalazine

Question: Which microbes would be necessary to empirically cover for in choosing an antibiotic treatment for uncomplicated cellulitis?

Answer: Streptococcus and Staphylococcus. Rapid Review Cellulitis Patient will be complaining of pain, redness, swelling PE will show tenderness, erythema with poorly demarcated borders, lymphedema Most commonly caused by Staph aureus and Streptococci

Question: At what gestational age should pregnant women with Herpes simplex virus begin suppressive therapy to reduce the likelihood of lesions during labor?

Answer: Suppressive therapy should be initiated at 34-36 weeks of gestation. Rapid Review Behçet's Syndrome Patient will be complaining of recurring genital and oral ulcerations, and relapsing uveitis PE will show painful genital and oral ulcers with a necrotic center and surrounding red rim

Question: Osteonecrosis is more likely with fractures of the scaphoid in the proximal or distal pole?

Answer: The more proximal the fracture, the greater likelihood of osteonecrosis. Rapid Review Scaphoid Fracture FOOSH Snuff box tenderness Possibly negative radiographs Complication: avascular necrosis Thumb spica splint

Question: True or false: Physician Assistants are mandated reporters of child abuse.

Answer: True.

Question: What is the imaging test of choice if a large or loculated effusion is suspected?

Answer: Ultrasonography

Question: What diagnosis does a positive head impulse test point to in a patient with continuous vertigo?

Answer: Vestibular neuritis. Rapid Review Peripheral Vertigo CN VIII, vestibular apparatus Onset: sudden Hearing loss, tinnitus Positional Nystagmus: unidirectional, never vertical, fatigable, can be inhibited No neurologic sx

Correct Answer ( D ) Explanation: Ulcerative Colitis (UC) is one of two inflammatory bowel diseases, the other being Crohn's disease. UC is a chronic idiopathic inflammatory bowel disease with recurrent symptoms and significant morbidity. The clinical presentation is highly variable and can mimic other types of disease such as pseudomembranous colitis and other infectious diseases which should be ruled out by stool studies as in the above scenario. The hallmark of UC is bloody diarrhea. Other symptoms include abdominal pain which is often times cramping in nature, tenesmus and fecal urgency. Patients can also present with anemia secondary to bleeding. Ulcerative Colitis is limited to the mucosal lining of the colon and always involves the rectum and spares the proximal gastrointestinal tract. In ulcerative colitis it is characteristic to have periods of symptomatic flare-ups and remissions. The goal of treatment in UC is to get flares into remission as well as provide relief of symptoms. First line treatment for mild to moderate UC is 5-aminosalicylic agents. These agents achieve clinical improvement in 50-70% or patients and remission in up to 30% of patients. Unlike UC, the depth of inflammation in Crohn's disease (B) is transmural as opposed to mucosal. Also in Crohn's the entire gastrointestional tract can be affected as opposed to the limited area affected in UC, which is often, times referred to as left sided colitis. Irritable bowel syndrome (IBS) (C) has symptoms of abdominal pain, cramping or bloating with episodes of diarrhea or constipation for at least 3 days out of the month for the last three months. The course of the disease varies in severity from person to person. IBS is not an inflammatory bowel disease and as such does not affect the colon wall in the same way as ulcerative colitis. Colon cancer (A) is a malignant tumor of the colon that often begins as a polyp. There are no clear signs and symptoms in the early stages of colon cancer which is why screening is recommended. Ulcerative colitis is a risk factor for colon cancer.

Answer: p-ANCA (50%-70% of cases). ASCA (10%-15% of cases). Rapid Review Ulcerative Colitis Patient will be 15-30-year-old Complaining of bloody diarrhea, crampy abdominal pain, tenesmus PE will show continuous mucosal inflammation always involving the rectum Treatment is sulfasalazine, surgery is curative Complications: Toxic megacolon, ↑ colon cancer risk

A 3-year-old boy from South America presents to your office with his father for a well-child exam. While listening to his lungs you notice that his back is covered with circular lesions approximately 3-4 cm in diameter with central ecchymosis and petechiae. Which of the following is the next best step? Ask about traditional medicinal practices being used Begin workup for bleeding disorders Call child protective services Obtain a urinalysis

Correct Answer ( A ) Explanation: A number of cultural practices can mimic signs of child abuse. Cupping is a technique used in Latin America, Asia, the Middle East and Eastern Europe. An open-mouthed vessel is heated and applied to the skin. Practitioners believe this will pull out the ailments and the result is lesions that appear like circular burns. Familiarity with cultural practices and medical conditions that can mimic child abuse can help to determine the correct diagnosis, start appropriate treatment and avoid unnecessary reports to child protective services. Bleeding disorders such as von Willebrand disease, factor VIII and IX deficiency or immune thrombocytopenia can present similarly to bruising caused by child abuse. Workup for bleeding disorder (B) may be appropriate after a complete history including questions about cultural practices is completed. Child protective services (C) should be called once the diagnosis of child abuse has been made and differential diagnoses have been ruled out. When in doubt, involve an interdisciplinary team to help determine the best course of action. Urinalysis (D) should be obtained when the history and physical point to abdominal trauma, dehydration or renal failure as the cause of the lesions.

A one-year-old boy of Mediterranean descent is found to have mild anemia on routine blood work. Further workup confirms the presence of beta-thalassemia trait. Which of the following is true regarding beta-thalassemia trait? If both parents have the trait, the risk of having a child with beta-thalassemia major is 25 percent It is inherited as an autosomal-dominant trait Management is with chronic blood transfusions and iron chelation therapy Mean cell volume (MCV) is usually above 80 fL/cell

Correct Answer ( A ) Explanation: A patient with anemia who is from the Mediterranean region and has a family history of anemia should raise the possibility of beta-thalassemia trait. Beta-thalassemia trait is caused by a defect in the single beta-globin. It is inherited as an autosomal recessive trait and if both parents are affected, the risk of having a child with beta thalassemia major is 25 percent. There are few findings on physical exam except for mild pallor. Beta-thalessemia trait is characterized as a mild hypochromic microcytic anemia. Older children may even have hemoglobin levels very close to normal. The peripheral blood smear often shows a variation in the size and shape of the red blood cells, hypochromia and basophilic stippling. A quantitative hemoglobin electrophoresis is usually done for confirmation. There is no specific treatment as beta-thalassemia trait is a benign condition. Establishing the diagnosis is important to prevent unnecessary treatment for iron deficiency. Beta thalassemia is inherited as autosomal-recessive (B). It is a mild microcytic anemia and the MCV is usually normal or slightly less than 80 fL/cell (D). Chronic blood transfusions and iron chelation therapy (C) is management in patients with beta thalassemia major.

Which of the following tests is recommended for routine HIV screening? Fourth generation combination HIV-1/2 immunoassay HIV-1 Immunofluorescence assay HIV-1 Western blot HIV-1/HIV-2 antibody differentiation immunoassay

Correct Answer ( A ) Explanation: Approximately 16% of individuals in the United States infected with HIV are not aware of their HIV status. Routine screening for HIV is now recommended for all individuals aged 15 to 65 years old. New recommendations from the Centers for Disease Control are to use a fourth generation combination HIV-1/2 immunoassay as the initial screening test. Use of this test allows for detection of HIV sooner after infection with the virus than with previous tests. Previous recommendations for a laboratory diagnosis of HIV infection were to use the HIV-1 Immunofluorescence assay (B) and HIV-1 Western blot (C) tests. The fourth generation combination HIV-1/2 immunoassay has replaced these tests due to being more accurate, having a faster turnaround time and fewer indeterminate results. A positive result with this test requires confirmation of HIV infection with the HIV1/HIV2 antibody differentiation immunoassay (D).

A 74-year-old man is having a pre-operative ECG performed. What is your interpretation of his ECG? Atrial fibrillation Atrial flutter Normal sinus rhythm Sinus tachycardia

Correct Answer ( A ) Explanation: Atrial fibrillation is an irregularly irregular rhythm due to uncoordinated atrial activation and random occurrence of ventricular depolarization. The atria are not contracting but they do discharge electrical impulses to the ventricles. However, no single impulse depolarizes the atria completely, so only an occasional impulse gets through the AV node. It is the most common sustained dysrhythmia in clinical practice. Atrial flutter (B) is a rapid atrial rhythm but due to nodal delay, ventricular response rate is slower. Therefore, atrial flutter always occurs with some sort of AV block so that not all impulses are conducted. The resulting block is often variable (2:1, 3:1, 4:1). P waves have characteristic sawtooth pattern. In normal sinus rhythm (C) and sinus tachycardia (D) the SA node is the pacemaker that causes the atria to depolarize regularly and, thus, the ventricles to depolarize regularly. Therefore, the ECGs for both of these rhythms have P waves and QRS complexes that occur regularly. The difference between these two rhythms is with the rate. The rate of sinus rhythm is 60-99. The rate of sinus tachycardia is >100.

A 37-year-old man with a past medical history of type 2 diabetes mellitus presents with complaints of pain and increasing redness and swelling of his right lower leg. He reports fever and chills. Physical exam reveals a temperature of 101° F with local tenderness, erythema with indistinct flat borders and moderate edema involving the right lower leg. Inguinal lymph nodes are non-palpable and there is no evidence of lymphatic streaking. Which of the following is the most likely diagnosis? Cellulitis Erysipelas Mucocutaneous candidiasis Necrotizing fasciitis

Correct Answer ( A ) Explanation: Cellulitis is characterized by an expanding, erythematous, edematous and tender lesion often involving the lower extremity. The borders of the area involved are not elevated and not demarcated. Borders of erythema should be marked in order to track spread of infected tissue. Pain, chills, and fever are commonly present. Diabetes mellitus is one of the risk factors for development of this disease. Delay in appropriate prompt antibiotic treatment may result in septicemia. Erysipelas (B) is a form of cellulitis that is more superficial, often involves the face, and is caused by beta-hemolytic streptococci. Unlike cellulitis, erysipelas has margins that are raised and more clearly demarcated. Lymphatic involvement evident by lymphatic streaking is also a prominent finding. While candidiasis (C) is frequently seen in patients with diabetes, the rash is seen as superficial, beefy red areas in intertriginous areas such as the groin. The rash of candidiasis does not demonstrate pain or heat upon palpation, unlike cellulitis and erysipelas. Necrotizing fasciitis (D) may be mistaken for cellulitis, especially in the early stages. It should be suspected in a patient who is toxic appearing and has bullae, crepitus or anesthesia of the involved skin with an underlying skin necrosis.

A 25-year-old truck driver presents with a 1-day history of throbbing rectal pain. Your examination shows a large thrombosed external hemorrhoid. Which one of the following is the preferred initial treatment for this patient? Elliptical excision of the thrombosed hemorrhoid Infrared coagulation Rubber band ligation of the hemorrhoid Stool softeners and a topical analgesic

Correct Answer ( A ) Explanation: Hemorrhoids are swollen blood vessels in the lower rectum. External hemorrhoids occur distal to the dentate line and develop as a result of distention and swelling of the external hemorrhoidal venous system. Engorgement of a hemorrhoidal vessel with acute swelling may allow blood to pool and, subsequently, clot; this leads to the acutely thrombosed external hemorrhoid. A patient with a thrombosed external hemorrhoid usually presents with complaints of an acutely painful mass at the rectum. Pain caused by hemorrhoids usually arises only with acute thrombus formation. This pain peaks at 48-72 hours and begins to decline by the 4th day as the thrombus organizes. The appropriate management of a thrombosed hemorrhoid presenting within 72 hours of the onset of symptoms is elliptical excision. Infiltration of a local anesthetic containing epinephrine is followed by elliptical incision and excision of the thrombosed hemorrhoid, its accompanying vein, and overlying skin. The elliptical incision allows the thrombosed hemorrhoid to be unroofed. Simple incision and clot evacuation is inadequate therapy and should not be performed. Incision and clot removal may provide inadequate drainage, resulting in rehemorrhage and clot reaccumulation. Rubber band ligation (C) is an excellent technique for management of internal hemorrhoids, and infrared coagulation (B) is also used for this purpose. Banding an external hemorrhoid would cause exquisite pain. If the pain is already subsiding or more time has elapsed, and if there is no necrosis or ulceration, measures such as sitz baths, bulk laxatives, stool softeners, and local analgesia may be helpful (D). Counseling to avoid precipitating factors such as prolonged standing, sitting, constipation, and delay of defecation is also appropriate.

At what age is imaging recommended for the evaluation of gross hematuria? 40 years 50 years 60 years 65 years

Correct Answer ( A ) Explanation: Microscopic hematuria is frequently encountered on urinalysis, and is often an incidental finding. In most cases, the cause of the microscopic hematuria is not determined and resolves spontaneously. In some cases, it may indicate underlying pathology (eg, kidney stone, infection) although most causes are benign. Gross hematuria, however, is frequently associated with an underlying malignancy. The risk of malignancy increases in persons over the age of 40 and increases even further over the age of 60 years. The positive predictive value of gross hematuria in men over 60 is 22.1% and 8.3% in women over 60. As little as 1 ml of blood in 1 L of urine can lead to gross hematuria. In addition to malignancy, specifically bladder and renal, other causes of gross hematuria include abdominal trauma, abdominal aortic aneurysm, glomerulonephritis, infection, BPH, recent bladder catheterization, ureteral or renal stone, drug or toxin induced. Patients with risk factors for significant disease (eg. age over 40 years, smokers, abdominal pain, trauma, or persistent hematuria) should undergo imaging. Choice of imaging (CT with IV contrast, stone protocol noncontrast CT, or renal ultrasound) depends in part on history and physical clues that may suggest a particular diagnosis. Cystoscopy is generally an outpatient procedure that can be done if imaging does not provide a diagnosis. 50 years (B), 60 years (C) and 65 years (D) are not in line with recommendations for evaluation of gross hematuria.

A 43-year-old HIV positive man presents with a 3-day history of painless, confluent white plaques, adherent to the oral and pharyngeal mucosa. The plaques reveal a raw, bleeding mucosa when scraped with a tongue depressor. What is the most likely diagnosis? Candidiasis Kaposi's sarcoma Leukoplakia Oral lichen planus

Correct Answer ( A ) Explanation: Oral candida or "thrush" is characterized by painless, confluent white plaques that can be removed with scraping. Candidiasis is often the first manifestation of HIV infection and should be considered in patients with no known predisposing cause of Candida overgrowth. Definitive diagnosis is made by wet preparation using potassium hydroxide revealing oval budding cells and hyphae of Candida albicans. Effective antifungal therapy may be achieved with oral fluconazole, clotrimazole troches, or nystatin mouth rinses. HIV patients may require longer courses of fluconazole of oral itraconazole. Kaposi's sarcoma (B) largely occurs with HIV infection as an AIDS-defining illness. These characteristic lesions of Kaposi's sarcoma are red or purple plaques or nodules on cutaneous or mucosal surfaces. The lesions of leukoplakia (C) present as white lesions commonly on the lateral aspects of the tongue that cannot be rubbed off. Leukoplakia is caused by the Epstein-Barr virus. Oral Lichen Planus (D) lesions are violaceous, flat-topped, angulated papules with very fine white streaks (Wickham striae) on the surface of the lips, tongues, and buccal mucosa. The mucosal lesions of Lichen Planus have a lacy white network overlying them that may be confused with leukoplakia. Oral Lichen Planus require biopsy and often direct immunofluorescence for diagnosis.

A 55-year-old man presents with new and insidious onset groin pain that began 5 months ago. His medical history includes sickle cell anemia. He currently abuses alcohol. Examination reveals painful hip rotation and decreased range of motion. There is no midline pelvic pain with resisted hip adduction. Pelvic radiographs reveal mild subchondral sclerosis about the femoral head, but no joint space narrowing or osteophytosis. Which of the following is the most likely diagnosis? Avascular osteonecrosis Legg-Calve-Perthes disease Osteitis pubis Osteoarthritis

Correct Answer ( A ) Explanation: Osteonecrosis of the femoral head is usually caused by a traumatic disruption of its blood supply or poor circulation due to other diseases. It most commonly occurs bilaterally and in those aged 40-60 years. Risk factors include a history of hip dislocation or femoral fracture, rheumatoid arthritis, systemic lupus erythematosus, Crohn's disease, chronic steroid or alcohol use, myeloproliferative disorders and sickle cell disease. Insidious onset of deep groin, hip or buttock pain predominates, however, acute presentations occur with acute collapse of the necrotic femoral head. Examination typically reveals antalgic gait and decreased and painful hip flexion, rotation and abduction. Radiographs early in the disease may be normal, or may reveal the earliest radiographical sign of this disease, subchondral femoral head sclerosis. MRI is necessary when the diagnosis is strongly suspected and the initial radiographs are normal or only positive for subchondral femoral head sclerosis. Pre-collapse treatment includes risk factor modification, physical therapy and pain management. Hip arthroplasty is typically reserved for function limiting head collapse. Secondary osteoarthritis is a common disease complication. Although Legg-Calve-Perthes disease (B) shares similar etiology and disease pathology with hip avascular osteonecrosis, its onset is in childhood, not adulthood. Osteitis pubis (C) is inflammation of the pubic symphysis which typically occurs in cumulative overuse of the hip adductors (as in certain athletes). Midline pelvic pain, with or without radiation to the groin or hip, that is worse with resisted hip adduction is a strong indicator of this condition. The classic radiographic findings of an osteoarthritic joint are subchondral sclerosis, cyst formation, joint space narrowing and osteophytes. The subchondral sclerosis of hip osteoarthritis (D) typically occurs in both the femoral head and acetabulum.

You finish taking a history of a patient at risk for suicide. Which of the following would suggest the highest likelihood of committing suicide? Age over 85 years Cohabitation with parents Married with children Tactile hallucinations

Correct Answer ( A ) Explanation: Suicide is the tenth most common cause of death in the US, and is in the top 3 most common causes of death in adolescents and young adults. It is more common in non-Hispanic White and American Indian men between the ages of 45-59 years. The greatest suicide rate is in elderly white males over 85 years old (A). Frequently, the primary care provider needs to assess the risk of suicide. Activities associated with committing suicide include buying a rope or firearm, filling out a will, contacting old friends and writing a suicide note. Common characteristics of those who commit suicide include a sense of violation, a preoccupation with death, a lack of humor, distractibility, few friends or family members, hopelessness and an inability to help themselves or a sense that there is nothing to help their condition. When evaluating for suicide risk, identify any of the above characteristics. Also, determine if suicidal ideation (thoughts of hurting oneself) is present, and if so, determine if a suicide plan is in place. The more detailed the plan, the higher likelihood of suicide. Also, determining the purpose of suicide can help to gauge the seriousness of the action. Any question of suicide must be coupled to an inquiry into the possibility of homicide, as both situations represent aggression. If there is a tendency toward aggression towards others (homicide), the likelihood of aggression toward oneself (suicide) is high. Furthermore, the presence of any of the following supports an actual committal of suicide: definite plan, activities one does before dying (saying goodbye to friends), family history of suicide, possession of a firearm, current substance use, depression of any type, anxiety of any type, command hallucinations, recent discharge from a psychiatric hospital, severe immediate loss such as an unexpected divorce, or isolation (patient is alone). Living with one's parents (B) and being married (C) are associated with a lower risk of suicide. Command, not tactile (D), hallucinations are associated with actual suicide.

Which of the following tests is the most accurate physical examination maneuver to identity an injury to the anterior cruciate ligament? Lachman's test McMurray's test Thompson test Valgus stress

Correct Answer ( A ) Explanation: The anterior cruciate ligament (ACL) is one of the four major ligaments of the knee (anterior cruciate, posterior cruciate, medial collateral, and lateral collateral). It provides anterior stability to the knee joint and is the most commonly injured ligament of the knee. The Lachman's test is the most accurate test for the ACL. In this test, the knee is flexed to 20 or 30 degrees and the thigh is stabilized. The tibia is then pulled anteriorly to look for abnormal excursion. McMurray's test (B) is used to identify meniscal tears. It is performed with the patient lying supine on the stretcher at which time the examiner flexes the foot and extends the knee while internally and externally rotating the tibia on the femur. A clicking indicates a positive test. The Thompson test (C) is performed to evaluate the integrity of the Achilles tendon. The patient's calf is squeezed which causes plantarflexion of the ankle joint. If flexion does not occur with the squeezing, the Achilles tendon is injured. Valgus stress (D) applied to the knee joint assesses the integrity of the medial aspect of the joint including the medial collateral ligament.

An eight-year-old boy comes in your clinic for follow-up. Two days ago, he was diagnosed with pneumonia and started on amoxicillin-clavulanic acid. However, fever and cough persisted and is accompanied by decreased appetite, chest pain, and difficulty breathing. You suspect a complicated pneumonia. Which of the following physical exam findings is consistent with pleural effusion? Dullnesss to percussion Early inspiratory crackles Trachea deviates away from affected side Vesicular breath sounds

Correct Answer ( A ) Explanation: The boy has history of pneumonia that failed treatment and developed a paraneumonic effusion. Bacterial pneumonias can be associated with complications involving the respiratory tract, which include pleural effusion, empyema, pneumatocoeles, necrotizing pneumonia, and lung abscesses. The incidence of paraneumonic effusions appears to be increasing in the United States. The causative organisms have changed over time with Streptococcus pneumoniae as the predominant organism. In addition, community-acquired Staphylococcus aureus is also becoming more common. The presentation of pleural effusion or empyema may depend upon when the child presents for medical attention. Some children present with symptoms related to empyema while others have been seen earlier in the course and appropriately treated for pneumonia but fail to respond. On physical examination, children may appear ill or occasionally be toxic-appearing. The majority are tachypneic, with fever and cough present in approximately 90 percent of patients. Chest examination may reveal a small degree of new scoliosis that is related to the patient's splinting toward the affected side. There may also be dullness to percussion, decreased air exchange, and possibly a pleural rub, on the side of the fluid collection. Early inspiratory crackles (B) is an exam finding that suggests small airway disease like bronchiolitis. Trachea deviating away from affected side (C) is a finding seen in tension pneumothorax. Vesicular breath sounds (D) is a normal physical examination finding.

A 28-year old woman presents with several painful ulcers she has developed in the vaginal area. Examination reveals multiple 0.5 cm to 1.5 cm oval ulcers with sharply defined borders and a yellowish-white membrane. She denies recent sexual activity. Except for recurring aphthous ulcers of her mouth, her past history is unremarkable. At this time, which of the following is the most likely diagnosis? Behçet's syndrome Herpes simplex Reactive arthritis Syphilis

Correct Answer ( A ) Explanation: The original description of Behçet's syndrome included recurring genital and oral ulcerations and relapsing uveitis. The genital and oral ulcers are classically painful with a necrotic center and surrounding red rim. Behcet's syndrome is more common in Japan, Korea, and the Eastern Mediterranean area, and affects primarily young adults. The cause is unknown. Two-thirds of patients will develop ocular involvement that may progress to blindness. Patients may develop arthritis, vasculitis, intestinal manifestations, or neurologic manifestations. This disease is diagnosed based on physical exam and clinical symptoms. Treatment of Behçet syndrome must be tailored to each patient's clinical manifestations. Corticosteroids are considered palliative; they are useful in controlling acute manifestations. Cytotoxic medications are usually indicated in patients with ocular, CNS, and vascular disease. Decreasing morbidity and mortality is the goal of treatment for children with Behçet syndrome. Reactive arthritis (C), formerly known as Reiter's syndrome is not associated with genital ulcers. Reactive arthritis is an aseptic inflammatory polyarthritis that usually is preceded by nongonococcal urethritis or infectious dysentery. The classic description of primary syphilis (D) is a solitary non-tender genital chancre. Syphilis does not present with oral ulcers or uveitis. Herpes simplex virus (B) is responsible for painful vesicles that are uniform in size. These lesions usually crust and then re-epithelialize and heal without scarring. The lesions are prominent and are often present internally on the mucosal surface of the oral or genital area.

A 67-year-old woman who was recently diagnosed with breast cancer presents to the ED accompanied by family who states that she has been increasingly confused over the past three days. Additionally, over the past week she has been complaining of fatigue, diffuse body aches, poorly localized abdominal pain, and constipation. Which of the following electrolyte abnormalities is the most likely explanation for her symptoms? Hypercalcemia Hyperkalemia Hypocalcemia Hypokalemia

Correct Answer ( A ) Explanation: The patient has hypercalcemia of malignancy. Total body calcium levels are tightly regulated by a feedback loop in which parathyroid hormone induces the bones and kidneys to increase serum calcium levels. Conversely, elevated calcium levels inhibit parathyroid hormone release. Vitamin D also plays a role in regulating calcium absorption in the gut. Hypercalcemia is defined as a serum calcium > 10.5 mg/dL. Symptoms of hypercalcemia are varied and include non-focal abdominal pain, constipation, fatigue, diffuse myalgias, anorexia, nausea, and vomiting. Neuropsychiatric manifestations such as depression, anxiety, and hallucinations may also be seen. Severe cases may lead to altered mental status, seizures, and coma. The mnemonic "moans, groans, stones, and psychic overtones," is useful for remembering hypercalcemia, including the above manifestations as well as a predilection for forming kidney stones. Hypercalcemia has numerous causes, including primary hyperparathyroidism, hyperthyroidism, multiple myeloma, cancer metastatic to bone, granulomatous disease (e.g. sarcoidosis). Medications, such as thiazide diuretics and estrogens, can also raise serum calcium levels. In this elderly patient with a recent diagnosis of breast cancer, bony metastases is a likely cause of hypercalcemia. Hyperkalemia (B) is often asymptomatic at lower levels, and at higher levels can cause dysrhythmias, cardiovascular collapse, and sudden cardiac death. Hypocalcemia (C) presents with muscle cramping and perioral and paresthesias of the fingers. Chvostek's and Trousseau's signs may be positive. Hypokalemia (D) is often asymptomatic, but can present nonspecifically with fatigue and muscle pain, but does not typically cause altered mental status. Hypokalemia often occurs as a result of a GI illness with vomiting and diarrhea, or as a result of diuretics.

A student health volunteer presents for evaluation after a needle stick. You review the labs of the source patient which are as follows: Anti-HAV negative; HBsAg positive; HBsAb negative; anti-HBc IgM positive, Anti-HCV positive. What is the most appropriate explanation of these labs? Active infection with hepatitis B Previous exposure to hepatitis A Previous infection with hepatitis B and C Vaccination against hepatitis B

Correct Answer ( A ) Explanation: The source patient is HBsAg positive (the surface antigen) and HBsAb negative indicating an active hepatitis B infection. The presence of surface antigen (HBsAg) indicates that the patient has an active infection with hepatitis B, either acute or chronic. The presence of anti-HBc IgM indicates that the infection is acute. Previous exposure to hepatitis A (B) would lead to a positive antibody test to the virus (Anti-HAV). This may also be present if a person has been immunized against the virus. If the patient had previous infection with hepatitis B and C (C), the patient would not have an active HBsAg test. Once the infection is cleared, the antigen test becomes negative and only the antibody tests will be positive (HBsAb and Anti-HCV). In cases where a person has been immunized against hepatitis B (D), the HBsAb test will be positive, but serology for HBsAg will be negative.

A G1P0 27-year-old woman at 12 weeks gestation presents to the Emergency Department with the above physical exam finding after hiking through the woods in Wisconsin. What is the most appropriate therapy? Amoxicillin 500 milligrams orally three times daily Ceftriaxone 1 gram intravenously every 12 hours Doxycycline 100 milligrams orally two times daily Rifampin 600 milligrams orally once daily

Correct Answer ( A ) Explanation: This patient is exhibiting erythema migrans, a hallmark of Lyme disease. Lyme disease is the most common vector-borne disease in the United States. It is endemic to New England, the mid-Atlantic states, and the upper Midwest. It is caused by the spirochete Borrelia burgdorferi and transmitted by the Ixodes dammini tick, more commonly known as the deer tick. The tick must be attached for more than 48 hours for transmission to occur. There are three stages of clinical Lyme disease. Early Lyme disease is characterized by erythema migrans, an erythematous blanching patch than may have central clearing and classically has a "bull's eye" appearance. Hematogenous spread leads to diffuse erythema migrans, which spares the palms and soles. Acute disseminated Lyme disease occurs approximately four weeks after initial infection and can include meningoencephalitis, Bell's palsy (which may be bilateral), or carditis which often manifests with AV block. Late Lyme disease develops greater than one year after initial infection and includes chronic arthritis with or without chronic subtle encephalopathy. Only 50% of patients remember a tick bite; thus, diagnosis may be difficult. Erythema migrans is diagnostic; however, not all patients present with this finding. Initial screening involves ELISA testing with Western Blot and PCR to confirm the diagnosis. If the diagnosis is suspected, empiric treatment should be administered. Treatment for early Lyme disease and mild acute disseminated Lyme disease in pregnant women or children under the age of eight years is amoxicillin 500 mg PO three times daily. Ceftriaxone 1 g IV every 12 hours (B) would be appropriate for a patient with neurologic or cardiac manifestations of Lyme disease. Doxycycline 100 mg PO two times daily (C) would be the typical treatment of choice for early Lyme disease; however, tetracyclines are contraindicated in pregnant women and children due to risk for permanent staining of the teeth during development. Rifampin 600 mg PO once daily (D) would be an appropriate alternative to treat ehrlichiosis if a patient had a history of anaphylaxis to tetracyclines.

Medical research has provided evidence to support which of the following as beneficial in reducing the risk of developing cataract? Corticosteroids HMG-CoA reductase inhibitors Sunglasses Vitamin K

Correct Answer ( B ) Explanation: A cataract is defined as a progressive clouding (opacification) of the lens that leads to painless blurred vision. Symptoms are progressive, developing over months or years. It is caused by oxidative damage to the lens, leading to increased deposition of insoluble proteins in otherwise transparent tissue. It is further classified as partial or complete, stationary or progressive or hard or soft. On exam, lens opacities can be grossly visible or seen as a diminished red reflex. The most common cause is aging, with the most common type being nuclear sclerosis affecting the central portion of the lens. Other etiologic classes are radiation, traumatic, metabolic, congenital and infectious. Outpatient surgery using local anesthetic is the most common treatment. Nearly 90% of patients treated surgically will gain 20/40 vision or better. Preventative tactics are controversial. There is limited, weakly positive, evidence that lutein supplementation is protective. There is evidence that supports regular statin (HMG-CoA reductase inhibitors) medication use decreases the risk of nuclear sclerotic cataract. Long-term use of corticosteroids (A) is a well-known cause of cataracts. Using ultraviolet-B protecting sunglasses (C) has been routinely recommended by many practitioners. However, there is no evidence to support such action. It was once thought that Vitamin A, C and E would prevents cataract, however, there is evidence that they do not. There is also no evidence that beta-carotene supplementation prevents cataract formation. Vitamin K (D) has no such antioxidant effects.

An 8-month-old boy has a painless swelling on the right side of his scrotum since birth that gets worse with crying. Transillumination of this side of the scrotum reveals increased fluid and the child does not cry on palpation of the testicle which lies in its normal position. Based on these findings, which of the following is the most likely diagnosis? Epididymitis Hydrocele Testicular torsion Varicocele

Correct Answer ( B ) Explanation: A hydrocele is a collection of fluid that accumulates in the tunica vaginalis. Communicating hydroceles result when the upper processus vaginalis fails to be obliterated, leaving an open tract between the peritoneum and the scrotum. The tract is closed in non-communicating hydroceles. Most hydroceles are right sided. Hydroceles may be present at birth, but they usually are painless and worsen with crying or exertion. Hydroceles often resolve spontaneously by the age of 18 months. Examination reveals transillumination and enlargement of the scrotum. Patients with a hydrocele that has persisted for more than 1 year or who are older than 18 months should undergo ultrasonography to ensure that the hydrocele is not a reactive hydrocele caused by testicular tumor or inflammatory process. Testicular torsion (C) presents with acute scrotal pain and swelling, an elevated testicle, and absence of the cremasteric reflex. Testicular torsion has been reported in all age groups, from the developing fetus to the elderly, but is most common in adolescence. Epididymitis (A) is inflammation of the epididymis, which is located along the posterior aspect of the testicle. The most common cause is infectious, and the etiology varies by age. Patients present with a painful, edematous scrotum and tenderness at the epididymis. A varicocele (D) is a collection of venous varicosities of the spermatic veins in the scrotum caused by incomplete drainage of the pampiniform plexus. Left-sided varicoceles account for 85 to 95% of cases but varicoceles are rare in children younger than 10 years. Varicoceles do not transilluminate.

Which of the following patients with atrial fibrillation has the greatest risk for complications from anticoagulation therapy? A 64-year-old woman with hypertension and history of colon cancer A 66-year-old man with a history of hypertension, diabetes, and ethanol abuse A 79-year-old man with a history of congestive heart failure A 90-year-old otherwise healthy woman

Correct Answer ( B ) Explanation: Atrial fibrillation is associated with arterial thromboembolism and ischemic stroke; therefore, anticoagulation is recommended in most cases. The CHADS2 scoring system can be used to identify patients at high risk for stroke. Recent guidelines for the management and treatment of acute atrial fibrillation from the American College of Cardiology, the American Heart Association, and the European Society of Cardiology recommend that patients with more than one risk factor be started on warfarin and that patients with only one risk factor take either warfarin or aspirin. However, the CHADS2 scoring system does not account for adverse events associated with anticoagulation and contraindications to warfarin therapy, including alcoholism, recent trauma or surgery, active gastrointestinal bleeding, respiratory bleeding, genitourinary bleeding, or significant risk of falls. The 66-year-old man with multiple comorbidities and a history of ethanol abuse is at greatest risk for complications from anticoagulation in atrial fibrillation. All of the other patients (A, C, & D) in the clinical scenario are at lower risk for complications from anticoagulation.

A 53-year-old man works as a metal fabricator in a local tool-and-die shop. While he was grinding a piece of iron, he felt something hit his eye. He flushed his eyes three times, but still feels like there is something "in there". Examination reveals severe left eye conjunctival injection, excessive tearing and eyelid erythema. You decide to perform a slit lamp examination with blue filter light. Which of the following stains is used in combination with this examination? Ammoniacal silver Fluorescein Rhodanine Trichrome

Correct Answer ( B ) Explanation: Corneal abrasions occur when the corneal epithelium becomes disrupted. There are several causes, such as dry eyes, foreign bodies (fingernails, animal paws, chips of wood or metal and branches or leaves) and contact lens use. They usually heal on their own with no complications. The major symptoms are pain, foreign body sensation, photophobia, blurred vision, tearing, eyelid edema, conjunctival injection, difficulty opening the eye and multiple patient attempts of washing out the eye. If suspected, initial evaluation requires the use of fluorescein and a slit lamp. If ocular penetration or retained foreign body is suspected, then order an ocular CT or MRI, depending on the metal content of the object. Fluorescein sodium-impregnated filter paper is commonly used in staining the eye to identify defects in the cornea. "Fluorescein uptake" occurs in areas of defects. This is performed using a blue light that will show the fluorescein as yellow-green in the area of defect. Ammoniacal silver (A) is used in the study of fungal bone infections. Rhodanine (C) is used to study copper deposits in tissue, such as liver tissue in a patient with Wilson's disease. Trichrome (D) staining is a basic procedure used in the histological study of tissues under a microscope.

A 28-year-old man presents with 2 months of nonbloody diarrhea and abdominal pain. Stool culture is negative for an infectious etiology. Endoscopic examination reveals small and large bowel fissuring and cobblestoning. Microscopic examination shows a transmural inflammatory infiltrate, but no crypt hyperplasia or villous atrophy. Hydrogen breath testing is negative. Which of the following is the most likely diagnosis? Celiac disease Crohn's disease Lactose intolerance Ulcerative colitis

Correct Answer ( B ) Explanation: Crohn's disease is characterized by diarrhea, intermittent abdominal pain, fever and malaise. Associated laboratory abnormalities may include decreased albumin and hemoglobin, as well as iron, vitamin B12 and folate deficiencies. This inflammation occurs throughout the colonic wall, as compared to the mucosal-only inflammation that occurs in ulcerative colitis. Crohn's may occur in any part of the intestines, both large and small, as well as the colon. Colonoscopy, small bowel imaging and esophagogastroduodenoscopy are used in making the diagnosis of Crohn's. Typical endoscopic findings are cobblestoning, fissures, ulcers and nonfriable mucosa. Microscopic examination usually reveals transmural inflammation with noncaseating granulomas. Complications include malabsorption and weight loss, as well as fistula, abscess and stricture formation. Acute treatment includes corticosteroids, antibiotics and biologic medications. Remission medication options are azathioprine, 6-mercaptopurine, the aspirin derivatives, methotrexate and other biologics. Celiac disease (A) is also an inflammatory small bowel disease. However, its microscopic examination typically reveals crypt hyperplasia or villous atrophy. Lactose intolerance (C) is diagnosed with a positive hydrogen breath test. Furthermore, transmural inflammation is not associated with this condition. Ulcerative colitis (D) is limited to the large intestine and colon, while Crohn's typically affects these areas as well as the small bowel.

A 27-year-old woman presents to your office with complaints of depression and thoughts of suicide. She is interested in starting both counseling and medication to address her symptoms. Which of the following is the safest medication to consider prescribing? Amitriptyline Fluoxetine Nortriptyline Venlafaxine

Correct Answer ( B ) Explanation: Depression and suicidal ideation are common complaints seen in the primary care setting. Any patient who reveals having thoughts of suicide should be assessed further to determine more details about the thoughts of suicide, as well as intent and plan. Risk factors for suicide include psychiatric illness, history of previous suicide attempts, individuals who have never been married, previous or active military service, childhood abuse, family history of suicide, and access to weapons. Women attempt suicide twice as often as men, but men are three times more likely to be successful. Management of a patient who is suicidal includes risk factor reduction, managing the underlying cause, close monitoring, and follow up. Determination of the lethality of the patient's current medication regimen is part of the risk reduction process. Selective serotonin reuptake inhibitors (SSRIs) seem to be safer in the case of an overdose than other agents. SSRIs, such as fluoxetine, are therefore the agents of choice in the treatment of depression for patients who are potentially suicidal. Tricyclic antidepressants, such as amitriptyline (A) and nortriptyline (C), are lethal if taken in high doses and their use should be avoided in patients who are suicidal. Venlafaxine (D) is a serotonin norepinephrine reuptake inhibitor that is dangerous in overdose and should also be avoided in patients deemed a high risk for suicide.

A patient presents with excessive urination and thirst. A comprehensive metabolic panel is normal. Urinalysis is significant for a low specific gravity. Which of the following is the most appropriate next test to order for the evaluation of these symptoms? Cosyntropin stimulation test Desmopressin stimulation test Dexamethasone suppression test Octreotide suppression test

Correct Answer ( B ) Explanation: Diabetes insipidus is a disorder of antidiuretic hormone (ADH) function, either production and release (central) or action (nephrogenic). Excessive urination (polyuria) and extreme thirst (polydipsia) are the main symptoms. These symptoms also occur in diabetes mellitus (DM). However, in DM, the urine typically contains high levels of glucose or protein, whereas in DI there is no component of glucose dysregulation and the urine is free of glucose and protein. DI commonly presents with the signs of dehydration and hypernatremia. If DI is suspected in a patient who presents with polydipsia and polyuria, the initial work-up is used to distinguish DI from other causes. At the minimum, this includes a serum glucose, bicarbonate and calcium level, which typically can be covered with ordering a comprehensive metabolic panel. In DI, urinalysis will show low specific gravity, low osmolarity and dilute color. Once other causes, namely diabetes mellitus, are ruled-out, a fluid deprivation test or desmopressin stimulation test is used to distinguish between the different types of diabetes insipidus. A cosyntropin test (A) is used to diagnose and evaluate disorders of ACTH and cortisol, not ADH. A dexamethasone suppression test (C) is used to evaluate the different forms of Cushing's syndrome and other hypercortisolemic states. The octreotide suppression test (D) is used to determine the viability of somatostatin-analogue treatment in patients with different neuroendocrine disorders, such as acromegaly and other excess growth-hormone states.

A 27-year-old man presents to your office with a complaint of numbness and weakness in his feet and fingers that began 24 hours ago. He tells you that two weeks ago he had an upper respiratory infection, but is otherwise healthy. Physical exam reveals right-sided facial droop and absent deep tendon reflexes. Which of the following is the most likely diagnosis? Botulism Guillain-Barré syndrome Lyme Disease Multiple sclerosis

Correct Answer ( B ) Explanation: Guillain-Barré syndrome (GBS) is a post-infectious, immune-mediated polyneuropathy. The cardinal feature of GBS is progressive, ascending, symmetric muscle weakness that begins in the lower extremities. Two-thirds of patients with GBS report a viral or bacterial infection prior to the onset of symptoms, most commonly upper respiratory or gastrointestinal. Muscle weakness progresses over hours to days and may eventually affect the muscles of respiration. Diagnosis is initially made based on clinical presentation and may be confirmed with cerebrospinal fluid analysis or neurophysiology studies including nerve conduction studies or needle electromyography. Patients diagnosed with GBS need to be hospitalized for close monitoring until it is determined where the patient is in the course of the disease. Many patients develop respiratory failure and require admission to the intensive care unit. Supportive care is the most important treatment modality for patients with GBS. Patients should receive deep vein thrombosis prophylaxis, bladder and bowel care, physical therapy and emotional support as part of their management plan. Prognosis ranges from full recovery to death, based on severity of symptoms and progression of the disease. Botulism (A) is a neurologic disorder that causes acute neuroparalysis. The clinical presentation of botulism is different from GBS due to patients with botulism having symmetric descending muscle paralysis or weakness instead of the ascending symptoms found in GBS. The second stage of Lyme disease (C) has neurologic manifestations such as facial palsy, headache and meningitis. This follows the first stage, which involves the classic skin rash, erythema migrans, and a flu-like febrile illness. Multiple sclerosis (D) presents with sensory and muscular symptoms that occur months or years apart and affect different locations in the body.

A 40-year-old man presents to the office with his wife who states that she has noticed some personality changes and aggressive behavior that began a few months ago. He is now having brief irregular involuntary movement of his arms, is unable to focus, and is having trouble problem solving. His wife also describes excessive blinking and head thrusts. Which of the following conditions are you suspicious for? Alzheimer disease Huntington disease Parkinson disease Wilson disease

Correct Answer ( B ) Explanation: Huntington disease is a fully penetrant autosomal dominant neurodegenerative disorder caused by an expanded trinucleotide (CAG) repeat in the gene for the protein huntingtin. The prevalence is approximately 10 per 100,000. Symptoms include a combination of a movement disorder, psychiatric disturbances, and cognitive dysfunction. Early on, the movement disorder is predominantly chorea, but parkinsonism and dystonia develop later. Some patients, especially those with juvenile onset, have a more rapidly progressive akinetic-rigid and dystonic form (the Westphal variant). Psychiatric manifestations, which are universal but widely variable, include personality changes, impulsiveness, aggressive behavior, depression, and paranoid psychosis. These psychiatric symptoms may precede the motor manifestations, and psychotropic drug therapy may be incorrectly blamed for the subsequent development of the movement disorder. Cognitive changes result in progressive subcortical dementia with disturbed attention, concentration, judgment, and problem-solving that differs from the typical cortical dementia of Alzheimer's disease. Oculomotor dysfunction most often manifestes in difficulties with refixating the gaze and a resulting tendency to use blinks and head thrusts is common. The diagnosis is confirmed by genetic testing. Although signs of short-term memory loss and lack of attention can be seen in Alzheimer's (A), these patients do not exhibit chorea or tic like behavior. Also, Alzheimer's does not typically occur as early as age 40. Parkinson's disease (C) is characterized by bradykinesa, resting tremor, and cogwheel rigidity but does not have the psychiatric symptoms. Wilson's disease (D) is characterized by Kayser-Fleischer rings, tremor, lack of coordination, drooling, and signs of liver failure.

A 4-month-old baby presents to your office with symptoms of worsening constipation, poor feeding, listlessness, and generalized weakness for two weeks. Physical exam findings include temperature of 98.6°F, ptosis, poor head control and poor ability to suck. Which of the following is the most appropriate initial therapy? Bisacodyl Human-derived botulism immune globulin Magnesium citrate Senna

Correct Answer ( B ) Explanation: Infant botulism, while rare, is the most common form of botulism seen in the United States. Raw honey is a frequently mentioned etiology of infant botulism. Other causes include corn syrup and soil and vacuum dust. The initial clinical manifestation is constipation, followed by motor function symptoms such as ptosis, facial and generalized weakness. Clinical presentation and electromyography findings consistent with infant botulism allow for a presumptive diagnosis while confirmatory stool studies are pending. Treatment in infants under age 1 year is with human-derived botulism immune globulin (BabyBIG) and should be administered as early as possible in the course of the infection. Equine-derived heptavalent botulinum antitoxin is used in non-infant cases of botulism. Bisacodyl (A) and Senna (D) are stimulant laxatives that are not recommended for children under the age of 3 years. Cathartics containing magnesium, such as magnesium citrate (C) should not be used in the treatment of infant botulism.

Which of the following oxygen delivery methods provides the highest fraction of inspired oxygen? Nasal cannula Non-rebreather mask Simple face mask Venturi mask

Correct Answer ( B ) Explanation: Many emergency department patients require supplemental oxygen. Several methods of oxygen delivery are available which provide varying concentrations of oxygen. The choice of method depends on on the patient's clinical condition and oxygen requirement. Nasal cannulae deliver oxygen via prongs inserted into the patient's nostrils, and are capable of oxygen flow rates of 2-5 L per minute, resulting in a FiO2 of 20-50%. A simple face mask can deliver oxygen at flow rates of 6-10 L/min for an FiO2 of 40-60%, and is useful in patients who find nasal prongs irritating, have epistaxis, or other reasons nasal prongs are problematic. Flow rates are determined by the patient's respiratory rate and tidal volume. A venturi mask mixes oxygen with room air at a settable concentration, and can deliver accurate and constant flow rates up to 40% FiO2. A venturi mask is helpful in situations where it is desirable to avoid excessive oxygen administration, such as in a patient with chronic obstructive pulmonary disease. Finally, a non-rebreather mask uses an oxygen reservoir and is capable of delivering the highest concentration of oxygen. Oxygen flow rates approach 10-15 L/minute, delivering an FiO2 of up to 100%. A non-rebreather mask must be secured tightly to the face, and can be uncomfortable for patients. Patients requiring intubation are typically preoxygenated using a nonrebreather mask in order to optimize oxygenation. Noninvasive positive pressure ventilation via continuous positive airway pressure (CPAP) and bilevel positive airway pressure (BiPAP) are also useful in optimizing oxygenation prior to intubation, especially if adequate oxygen saturations cannot be achieved using conventional means. Nasal cannula (A), simple face mask (C) and venturi mask (D) provide an FiO2 that is lower than that provided by a non-rebreather mask.

You are performing a newborn exam on a one-day-old baby girl. In doing so, you hear a continuous, rough, machinery-like murmur during cardiac auscultation. Which of the following heart lesions is most closely associated with the murmur you heard? Atrial septal defect Patent ductus arteriosus Still's murmur Ventricular septal defect

Correct Answer ( B ) Explanation: The ductus arteriosus (DA) in utero shunts blood to the aorta, bypassing fetal lungs, and normally closes within the first five days of life. When a DA remains open longer than one week, it is considered abnormal and is referred to as a patent ductus arteriosus (PDA). This condition is more common in females than in males, and has an increased association with maternal rubella. Clinical findings include a loud, continuous, machinery-like murmur best heard at the left upper sternal border. In addition, infants with a PDA may show signs of labored breathing, bounding pulses, and tire easy with feeds. Closure of the patent ductus arteriosus is stimulated by administration of prostaglandin synthesis inhibitors, such as indomethacin or aspirin, which is effective in premature infants. Surgical correction is often required for term infants. An atrial septal defect (ASD) (A) involves incomplete formation of the atrial septum, located in the region of the foreman ovale. It involves a left to right shunt, and the majority of ASD's are asymptomatic. A soft, fixed, systolic ejection murmur, that is has a widely split S2, is heard at the left upper sternal boarder. Atrial septal defects can be treated conservatively and be left alone until the patient reaches age three, at which point surgical closure is recommended. Still's murmur (C) is the most common innocent murmur in childhood. It typically occurs in children from age two until six. The murmur associated with a Still's murmur is musical and vibratory in nature. It is early, systolic, high-pitched, and heard best at the heart's apex. Most importantly it increases with the patient in supine position and diminishes when the patient sits or stands. Ventricular septal defects (D) are the most common congenital heart disease. A loud, harsh, holosystolic murmur associated with this condition is heard best at the left lower sternal boarder of the heart. It also may be associated with a systolic thrill. Patients are often sluggish, feed poorly, and have difficulty gaining weight appropriately. Approximately 85% of VSD's close spontaneously, though if the patient continues to have poor growth patterns or pulmonary hypertension, surgical closure will likely be required to treat the condition.

A 16-year-old girl is in the clinic for a follow-up for a fever. She has been having prolonged fever accompanied by fatigue, night sweats, and loss of appetite. On physical examination, she has a temperature of 38°C with an enlarged cervical lymph node on the right that is painless, rubbery, and firm. There is no hepatosplenomegaly. She is then referred for excisional biopsy of the enlarged lymph node that shows Reed-Sternberg cells. Which of the following is the most likely diagnosis? Epstein-Barr virus infection Hodgkin lymphoma Metastatic adenopathy Mycobacterial infection

Correct Answer ( B ) Explanation: The girl has constitutional symptoms with an enlarged lymph node showing Reed-Sternberg cells. These findings are consistent with Hodgkin lymphoma (HL). HL is a malignant lymphoma that accounts for approximately 7 percent of childhood cancers and 1 percent of childhood cancer deaths in the United States. The incidence of HL in childhood varies by age such that it is exceedingly rare in infants, but is the most common childhood cancer in the 15- to 19-year-old age group. Most children with HL present with painless lymphadenopathy, which is usually cervical, supraclavicular, axillary, or inguinal. The affected lymph nodes typically feel rubbery and more firm than inflammatory adenopathy. They may be sensitive to palpation if they have grown rapidly. Patients with HL may present with nonspecific systemic symptoms, including fatigue, anorexia, and weight loss. Patients are classified to have "B" symptoms when they present with fever (> 38.0°C), drenching night sweats, and weight loss (≥ 10 percent loss within six months before diagnosis). The B symptoms have an important implication for staging and prognosis. The diagnosis of HL is established by histologic examination, usually by excisional biopsy of an enlarged lymph node that demonstrates malignant classic Reed-Sternberg cells or their variants. Epstein-Barr virus infection (A) can cause infectious mononucleosis with symptoms that include fever, pharyngitis, adenopathy, fatigue, and atypical lymphocytosis. Metastatic adenopathy (C) results from metastasis from a primary cancer which the patient does not have. Mycobacterial infection (D) can present with lymphadenopathy, especially in the neck (scrofula). The nodes are typically nontender, enlarge over weeks to months without prominent systemic symptoms, and can progress to matting and fluctuation.

You are seeing a 79-year-old woman who suffers from severe diabetic polyneuropathy. She is taking antihyperglycemic medications to maximize her glycemic control. In an effort to improve her care, you recommend a pain medication. Which of the following medications would be most appropriate to trial? Aspirin Gabapentin Glipizide Oxycodone

Correct Answer ( B ) Explanation: The goal of treating polyneuropathy is two-fold: reducing, removing or managing any underlying condition, and using neuropathic pain medications. Gabapentin substantially reduces the pain associated with polyneuropathy. It is generally well tolerated. A typical starting dose in the elderly population is 100 mg at night, with a gradual increase to twice a day and then three times a day over 1-2 weeks. Slow, further titration may be necessary to optimize pain control, but must be balanced with the possible side effects of sedation and dizziness. Tricyclic antidepressants, such as amitriptyline are also generally considered effective. Aspirin (A) is not used for on-going management of neuropathic pain. Glipizide (C) is a medication used in improving glycemic control. It is in the sulfonylurea class of diabetes medications, whose function is to stimulate the pancreas to produce insulin. Although tight glycemic control is recommended in any diabetic patient with polyneuropathy, it is not a pain medication. Opioid medications, such as oxycodone (D) should be reserved as a last option in treating any neuropathic pain condition. Some practitioners argue that opioids should not be used in the elderly due to the serious side effects of constipation and increased risk of falls.

You examine a three-year-old girl for possible drowning. She was unsupervised for a few minutes and later her mother found her by the pool. The mother called 911, and CPR was performed for five minutes. Upon arrival in the emergency room, she is unresponsive and is immediately intubated. Her blood pressure is 70/50 mm Hg, heart rate is 125 beats per minutes, and pulse oximetry reading is 95 percent. Which of the following is the correct statement regarding water safety? Adolescents who had swimming lessons are allowed to swim alone Fencing that surrounds a pool with a self-locking gate reduces the risk of drowning Less supervision is fine for toddlers who had swimming lessons Water wings and floaters can be relied upon as drowning prevention measures

Correct Answer ( B ) Explanation: The pediatrician has the opportunity to inform families at risk regarding water safety through anticipatory guidance. Prevention is the most effective way to decrease the injury burden of drowning. A family-centered approach to anticipatory guidance for water safety is to explore and identify the water hazards that each family is exposed to in their environment. The practitioner can then discuss the best tools and strategies for prevention that are relevant for the particular family. A common preventive strategy for exposure to all water types and all ages is ensuring appropriate supervision. Pediatricians should define for parents what constitutes appropriate supervision at the various developmental levels of childhood. Supervision of infants and young children means that a responsible adult should be with the child every moment. Another strategy to reduce the risk of drowning is with isolation fencing that completely surrounds a pool, with a secure, self-locking gate. In families who have a pool on their property, caregivers often erroneously believe that if a child falls into the water there will be a loud noise or splash to alert them. On the contrary, drowning events are usually silent, increasing the time until the child is discovered. This finding highlights the importance of the guideline that the fence actually separates the pool from the house, not just surround the entire property. Adolescents who had swimming lessons are allowed to swim alone (A) is wrong. Children and adolescents should never swim alone regardless of their swimming abilities. Even as they become more independent and participate in recreational activities without their parents, they should be encouraged to seek areas that are watched by lifeguards. Less supervision is fine for toddlers who had swimming lessons (C) is a wrong statement. Parents need to know that swimming lessons and acquisition of swim skills cannot be solely relied on to prevent drowning. Water wings and floaters can be relied upon as drowning prevention measures (D) is false. Young children should wear lifejackets that will float their head up and the use of toys like water wings and floaters are not reliable.

A previously healthy 17-year-old boy is brought to the ED by ambulance. He became very ill over the past few hours. His vital signs are T 39.4ºC, HR 142, BP 90/52, RR 20, and a pulse oximetry of 94% on room air. On exam, he has a stiff neck and the rash seen above on his leg. You establish an intravenous line, draw a blood culture, order a complete blood count and electrolyte assessment, and administer a fluid bolus. Which of the following is the most appropriate next step? Administer hydrocortisone to treat hemorrhagic adrenalitis (Waterhouse-Friderichsen syndrome) Administer intravenous ceftriaxone and vancomycin Perform a CT scan of the head Perform a lumbar puncture

Correct Answer ( B ) Explanation: This patient is presenting with severe meningococcal septicemia caused by the aerobic gram-negative diplococcus N. meningitidis, a natural organism living in the nasopharynx of humans, its only host. Patients with this infection may progress rapidly. The lumbar puncture may be deferred until the patient is stable and should not delay antibiotic administration. Although N. meningitides continues to be sensitive to penicillin, ceftriaxone and vancomycin should be administered to cover for N. meningitides and resistant pneumococcal meningitis, which may also cause purpura fulminans. Hemorrhagic adrenalitis is adrenal gland failure due to bleeding into the adrenal glands, caused by severe bacterial infection (most commonly N. meningitidis). The bacterial infection leads to massive hemorrhage into 1 or (usually) both adrenal glands. Treatment with antibiotics is the primary treatment. Hydrocortisone (A) can sometimes reverse the hypoadrenal shock. Although the patient's mental status change can indicate an elevated intracranial pressure from meningococcal meningitis inflammatory changes, his presentation with low blood pressure and tachycardia is more consistent with severe meningococcal septicemia. A head CT scan (C) is not indicated in patients with altered mental status due to shock from decreased brain perfusion. A lumbar puncture (D) should always be deferred if the patient has signs of airway difficulties, shock, elevated ICP, or coagulopathy until the patient has been fully stabilized.

A 44-year-old woman presents with a sensation that the room is moving. The symptoms started 1 hour ago. She states that when she turns her head to the left, she feels the spinning sensation and when she brings her head back to neutral it subsides. The episodes are accompanied by nausea and vomiting. Her neurologic exam is unremarkable. Which of the following is the most appropriate management for this patient's condition? CT scan of the head Epley maneuver Meclizine MRI of the brain

Correct Answer ( B ) Explanation: This patient is suffering from benign paroxysmal positional vertigo (BPPV) and should be treated with the Epley maneuver. Vertigo can either be of peripheral or central origin. Peripheral causes of vertigo are typically benign and include otitis media, labyrinthitis, Meniere's disease, vestibular neuronitis, and BPPV. Central causes of vertigo are, in general, dangerous and include CNS infections, vertebral basilar artery insufficiency, cerebellar hemorrhage, or infarction and temporal lobe epilepsy. Differentiating central from peripheral vertigo is vital in determining the etiology and course of action. Peripheral vertigo tends to be sudden and severe in onset lasting for seconds to minutes at a time. Nystagmus should be unidirectional and head position worsens symptoms. Central vertigo may be sudden or gradual in onset and can last for weeks or months. Nystagmus may be horizontal, vertical or rotatory and bidirectional. Head position does not affect the vertigo and there are usually associated neurological findings. BPPV is typically described as a severe onset of room-spinning, nausea and vomiting associated with a specific head position. The symptoms improve with returning the head to the neutral position. BPPV is typically caused by the presence of an otolith in one of the semicircular canals. In patients with suspected BPPV, a Dix-Hallpike maneuver can be performed to aid in diagnosis and an Epley maneuver can be performed to reposition or expel the otolith from the semicircular canal. CT scan of the head (A) may be helpful if a central cause of vertigo is suspected although CT scan usually does not provide adequate imaging of the posterior fossae. Meclizine (C) is useful in the treatment of continuous vertiginous symptom and not of episodic symptoms. MRI (D) is more useful than CT is assessing patients for central causes.

A 68-year-old man presents for a wellness visit. He is a former smoker, having quit 3 years ago. He has a 35 pack-year history of smoking. He has no other comorbidities and is feeling well at the time of his visit. What screening needs to be performed with regard to lung cancer? Chest X-ray Low dose chest CT scan No screening is indicated Pulmonary function tests

Correct Answer ( B ) Explanation: This patient needs annual low dose chest CT scans until he has been a non-smoker for 15 years consecutively. United States Preventive Services Task Force (USPSTF) guidelines state patients aged 55-80 years old who are either smokers or former smokers who quit within the past 15 years and have a total of 30 pack-years or more need annual low dose chest CTs until they have been non-smokers for 15 or more years. Lung cancer is the third most common cancer in the USA and the top cause of cancer related deaths. Smoking is the most common risk factor for lung cancer and is associated with 85% of diagnosed lung cancer in the USA. Incidence increases with age and cumulative smoke exposure. Low dose CT scanning has a sensitivity of 93.8% and a specificity of 73.4% when screening for lung cancer. Chest X-ray and pulmonary function tests (A, D) are not the proper screening tests for lung cancer. Screening (C) is indicated according to the United States Preventive Services Task Force (USPSTF) guidelines.

A 14-year-old boy presents with cracked, scaly skin in the interdigital webs of his feet. The remainder of his exam is benign. He recently began showering in the school locker room after football practices. What is the treatment of choice? Oral trimethoprim-sulfamethoxazole Topical clotrimazole Topical corticosteroids Topical nystatin

Correct Answer ( B ) Explanation: Tinea pedis, also known as athlete's foot, is a dermatophyte infection affecting the feet. The infection begins as vesicles or bullae on the toes or soles and may progress to significant scaling. Involvement of the interdigital webs is classic. Predisposing factors for tinea pedis include use of locker rooms and swimming pool facilities, where spores may be readily transmitted after shedding from infected individuals. Attacks are often self-limited but may recur after activities that involve significant sweating of the feet. Additionally, affected individuals may develop auto-eczematization, or an id reaction, in which papular eruptions occur over distant body sites. The etiology of id reactions is thought to be autoimmune. The diagnosis of tinea pedis can be microscopically confirmed with KOH preparation of scrapings of the affected skin. The treatment of choice is a topical antifungal, such as topical clotrimazole, for four weeks. Other options include other azoles, such as miconazole, or topical allylamines such as terbinafine or naftifine. Treatment of the primary infection also results in resolution of the id reaction. Notably, the cracked skin of tinea pedis is susceptible with Staph or Strep species, and thus signs of superinfection should be monitored. Oral trimethoprim-sulfamethoxazole (A) is not effective against dermatophytes. It is, however, an excellent treatment for suspected Staphylococcus aureus superinfection of the cracked skin of tinea pedis. Topical corticosteroids (C) are not recommended in the treatment of dermatophyte infections, as they may exacerbate the infection or alter its appearance, making a later diagnosis more challenging. Topical nystatin (D) is not effective against dermatophytes but provides excellent therapy for cutaneous candidal infections.

Which of the following physical exam findings is most suggestive of appendicitis? Kehr's sign Murphy's sign Rovsing's sign Scarf sign

Correct Answer ( C ) Explanation: Appendicitis is an inflammation of the vermiform appendix, which is located at the base of the cecum near the ileocecal valve. Appendicitis occurs when the lumen of the appendix is obstructed, generally by lymphoid hyperplasia, fecaliths, infections, or parasites. It most commonly affects individuals in the second and third decades of life. Symptoms of appendicitis include abdominal pain, anorexia, nausea, and vomiting. The abdominal pain generally begins as periumbilical pain that migrates to the right lower quadrant. Fever may occur later in the course of the illness. A number of physical exam signs may be elicited to help facilitate diagnosis. These include Rovsing's sign, which occurs when palpation of the left lower quadrant elicits pain in the right lower quadrant. Diagnosis is based on history and physical exam and is confirmed by imaging, either computed tomography (CT) or ultrasound. Treatment is surgical intervention and removal of the appendix. Kehr's sign (A) is referred pain to the left shoulder that worsens with inspiration in patients with splenic injuries. Murphy's sign (B) is elicited in the physical exam for suspected cholecystitis by instructing the patient to take a deep inspiration while palpating the right costal margin just beneath the edge of the liver. A positive Murphy's sign occurs when the patient experiences discomfort and catches their breath on the exhalation. Scarf sign (D) is used in the motor function evaluation of a newborn to determine range of shoulder adduction.

A 19-year-old man with refractory schizophrenia is brought to the ED by his mother for an acute psychotic episode. Routine laboratory tests reveal a WBC count of 1.0. His mother reveals that he recently started a new medication. Which of the following medications is the patient most likely taking? Chlorpromazine Clonazepam Clozapine Levetiracetam

Correct Answer ( C ) Explanation: Clozapine is an atypical antipsychotic medication that controls both positive and negative symptoms of psychotic disorders. Clozapine causes agranulocytosis in 1%-2% of patients during the first few months of treatment. The risk of developing agranulocytosis is highest approximately three months into treatment and decreases substantially thereafter. Other potential side effects include dyslipidemia, diabetes, and decreased seizure threshold. Levetiracetam (D) is an anticonvulsant agent used in the treatment of epilepsy. Levetiracetam is well tolerated. Some serious side effects may include depression, hallucinations, and suicidal thoughts. Chlorpromazine (A) is a typical antipsychotic used in the treatment of psychosis, schizophrenia, and recurrent hiccups. Its main side effects are due to its anticholinergic properties. Clonazepam (B) is a benzodiazepine that enhances the action of GABA and causes general CNS depression. It is not associated with agranulocytosis.

An 18-year-old hurdler gets tripped-up and falls, contacting the running-track through his hyperextended left wrist. He presents to the ED with dorsolateral wrist pain, erythema, edema and anatomic snuffbox tenderness. Orthopedic consultation is obtained. Initial AP, lateral and oblique radiographs are negative for fracture. He is splinted and discharged. His discharge paperwork would most likely contain which of the following directions? Obtain MRI in 2 weeks if pain persists Obtain primary-care medical clearance examination for upcoming orthopedic surgery Obtain repeat radiographs in 2 weeks Obtain rheumatological consultation for wrist inflammation

Correct Answer ( C ) Explanation: Falling On an Out-Stretched Hand (FOOSH injury; "outstretched" refers to wrist hyperextension) is a common cause of scaphoid fracture, which represents the most common fractured carpal bone. The most common fracture site on the scaphoid is the waist. Since the blood supply to the scaphoid enters distally, any >1 mm fracture-displacement is worrisome for being complicated by osteonecrosis and nonunion. A scaphoid fracture should be suspected any time there is snuff box tenderness. Wrist and scaphoid X-rays may be negative in the setting of acute fracture. Therefore, patients with snuff box tenderness or suspicion for scaphoid fracture should be placed in a thumb spica splint with repeat radiographs in 2-3 weeks. MRI (A) is considered after the repeat radiographs show no fracture despite symptoms persisting or if suspicion is still high. It is unknown if a true fracture is present at this time, as such, surgery (B) would not be set-up at discharge. This wrist erythema and edema is related to trauma, and not to an underlying rheumatologic condition (D).

A 27-gestational-week-old infant is delivered in the emergency department. The neonate is cyanotic, tachypneic and shows chest wall retractions. An emergent chest radiograph reveals a diffuse ground-glass appearance. Meconium aspiration is not suspected. Which of the following is the most likely diagnosis? Erythroblastosis fetalis Hydrops fetalis Infant respiratory distress syndrome Persistent pulmonary hypertension of the newborn

Correct Answer ( C ) Explanation: Infant (neonatal) respiratory distress syndrome is the new term for hyaline membrane disease. It is also known as respiratory distress syndrome of the newborn and surfactant deficiency disorder. It occurs in premature infants whose pulmonary system has not completely matured, namely those born before 34 weeks (especially between 26 and 28 weeks) gestational age, affecting about 1% of all newborns. It is due to a deficiency of surfactant, a lipid-protein substance produced by type II pneumocytes which decreases surface tension in, and prevents collapse of, alveoli. Symptoms include tachypnea, tachycardia, cyanosis, nasal flaring, expiratory grunting and chest wall retractions. Diagnosis is made clinically and radiographically. The typical radiograph shows decreased lung volume, air-bronchograms and a ground-glass appearance. Treatment includes oxygen, continuous positive airway pressure, exogenous surfactant and in severe cases, intubation and extracoporeal membrane oxygenation. Hemolytic disease of the newborn, also known as Rh disease, or Rh incompatibility, occurs in subsequent pregnancies of a Rh negative woman and RH positive fetus. Mild cases present as neonatal anemia and reticulocytosis, while severe causes result in erythroblastosis (A), hyrops fetalis or stillbirth. Hydrops fetalis (B) is an abnormal accumulation of edema in at least two fetal compartments, due to several maternofetal immune and nonimmune conditions. Its common manifestation is prenatal heart failure, not respiratory failure. Persistent pulmonary hypertension of the newborn (PPHN) (D) is due to a failure of the circulatory transition which occurs during birth, mainly due to a right-to-left intracardiac shunt. It occurs mainly in term or near-term neonates, and only very infrequently in preterm neonates. Lung fields are mostly clear in radiographs of those neonates diagnosed with PPHN. It is commonly caused by meconium aspiration.

Which of the following is true regarding Meckel's diverticulum? Commonly seen at five years of age Diagnosed by abdominal ultrasound Diagnosed by Meckel's scan using Tc99m pertechnetate Presents with bilious vomiting

Correct Answer ( C ) Explanation: Meckel's diverticulum usually presents with painless rectal bleeding and is due to incomplete obliteration of the vitelline duct. It is located two feet from the ileal cecal valve, is two inches long and commonly seen in children younger than two years of age. Bleeding results from ulceration of the ileal mucosa caused by acid secretion from the gastric mucosa-lined epithelium. It is diagnosed via Meckel's scan using Tc99m pertechnetate, which is a radioactive chemical taken up by the gastric mucosa. The treatment is surgical resection. Meckel's diverituculum is commonly seen in children younger than two years of age (A). Abdominal ultrasound (B) is not used in the diagnosis of Meckel's diverticulum. Meckel's diverticulum does not usually present with bilious vomiting (D) unless it is associated with bowel obstruction.

A 45-year-old Native American woman comes to the urgent care clinic with a 5-hour history of right upper quadrant pain, fever, nausea, vomiting, and anorexia. She says the pain radiates to her shoulder and back. She says she began experiencing these symptoms shortly after eating a hot dog and chili cheese fries. She smokes 2 packs of cigarettes per day, drinks 2-3 alcoholic beverages every night, and occasionally smokes marijuana. Her temperature is 38.8°C (101.8°F). Palpation of the abdomen shows voluntary guarding. An abdominal ultrasound shows a gallbladder wall of 6 mm. Which of the following contributed the most to the development of this patient's condition? Alcohol consumption Cigarette smoking Ethnicity Marijuana smoking

Correct Answer ( C ) Explanation: Right upper quadrant pain (radiating to the shoulder or back), fever, nausea, vomiting, and anorexia most likely suggests acute cholecystitis. Acute cholecystitis refers to a syndrome of right upper quadrant pain, fever, and leukocytosis (with a left shift) associated with gallbladder inflammation that is usually related to gallstone disease. Onset of symptoms generally occurs following ingestion of a fatty meal (as in this case). Several risk factors have been identified with the development of gallstones. They include female gender, advancing age (> 40), previous pregnancies, family history of gallstone disease, oral contraceptive pills (and estrogen replacement therapy), sedentary lifestyle (eg, obesity), and type 2 diabetes. Native Americans have the highest prevalence of gallstones, with the disease reaching epidemic proportions in this population. Although, alcohol consumption (A) is not directly related to the development of gallstones, liver cirrhosis, has been shown to be a major risk factor for gallstones. Cigarette smoking (B), and marijuana smoking (D) have not been shown to be directly associated with the increased risk of the development of gallstones.

Which of the following stable patients with back pain should have an emergent MRI performed? 23-year-old man with a positive straight leg raise and otherwise normal neurologic examination 30-year-old woman with back pain and dysuria 34-year-old man with a history of intravenous drug abuse with back pain and constipation 45-year-old woman with back pain after a car accident with no midline tenderness and normal neurologic examination

Correct Answer ( C ) Explanation: The majority of patients who present with back pain do not require emergent imaging. However, those who may have a serious cause of their back pain including cauda equina syndrome, epidural abscess, vertebral osteomyelitis and other causes of cord impingement should be considered for emergent imaging of the spine. Patients that present with back pain should be screened for "red flags" in their presentations that support one of these critical diagnoses. These red flags include age (both extremes), history of cancer (especially those known to metastasize to bone), fever, weakness, saddle anesthesia, change in bowel or bladder function (incontinence or retention/constipation), trauma, history of immunocompromise (including chronic steroid use) and intravenous drug abuse. Patients with these components to their presentations have a higher risk of dangerous diagnoses and should be considered for early imaging with either CT (better for bony abnormalities) or MRI (better for spinal cord visualization). A positive straight leg raise (A) (pain radiating from the back into the leg below the knee) is suggestive of a protruding disk but can be worked up as an outpatient. Patients with symptoms of pyelonephritis (B) rarely require imaging unless there is a more complicated picture to their presentation (presence of kidney stone, sepsis, septic shock). Patients with trauma but no midline tenderness and a normal neurologic examination (D) do not require imaging.

A 35-year-old man presents to the ED with a severe rash. A localized portion is seen in the image above. He states that he was cleaning away some brush from the woods behind his house a couple of days ago while only wearing shorts. On exam, you note similar lesions on his face, back, legs, arms, and chest. Which of the following is the most appropriate treatment? Cephalexin for 7 days Diphenhydramine as needed Prednisone taper over 21 days Prednisone taper over 7 days

Correct Answer ( C ) Explanation: The patient was exposed to poison ivy and developed allergic contact dermatitis. Contact dermatitis is an inflammatory reaction of the skin to a chemical, physical, or biologic agent. The inducing agent acts as an irritant or allergic sensitizer. Clothing, jewelry, soaps, cosmetics, plants, and medications contain allergens that commonly cause allergic contact dermatitis. The most common allergens include rubber compounds, plants of the Toxicodendron genus (poison ivy, poison oak, sumac), nickel (often found in jewelry), paraphenylenediamine (an ingredient in hair dyes and industrial chemicals), and ethylenediamine (a stabilizer in topical medications). Clinical presentation is variable with primary lesions being papules, vesicles, or bullae on an erythematous base. The distribution of the eruption depends on the specific allergen and may be localized, asymmetric linear, or unilateral. The classic lesion of poison ivy is a linear eruption that occurs from the person brushing against the poison ivy leaf. The rash usually appears 2-21 days after exposure and is associated with intense pruritus. Treatment for mild cases of contact dermatitis from poison ivy includes calamine lotion or a topical steroid and oral antihistamine. However, in moderate to severe cases, systemic steroids are indicated and should be continued for 2-3 weeks, with a gradual taper to prevent rebound of the disease. Systemic antihistamines can be taken concomitantly to help control the pruritus. Cephalexin (A) is a first-generation cephalosporin commonly used in the treatment of cellulitis. It is not used as a primary treatment in contact dermatitis but may be necessary if a secondary bacterial infection develops. Diphenhydramine (B), an antihistamine, is used as adjunctive therapy in severe cases and primary therapy for mild cases of allergic contact dermatitis. Prednisone (D) should be administered over 2-3 weeks rather than 7 days to avoid rebound of the disease. A 7-day course may treat the lesions temporarily, but often the rash returns.

A 32-year-old woman complains of back pain and difficulty urinating for five days. The pain is located in her lumbar spine and is constant and dull. She says she feels the urge to urinate every five minutes, but only a small amount of urine is expressed. On physical exam, her vital signs are T 38.4°C, BP 110/72 mm Hg, HR 115 bpm. Cardiac exam reveals a 2/6 systolic murmur. Lung exam is normal. Abdominal exam reveals a large, tender suprapubic mass. Musculoskeletal exam reveals tenderness over her lumbar spine and track marks consistent with IV drug use in her left antecubital fossa. Neurological exam reveals 4/5 strength in her bilateral lower extremities. What is the most likely cause of her symptoms? Bladder cancer Necrotic uterine leiomyoma Spinal epidural abscess Tubo-ovarian abscess

Correct Answer ( C ) Explanation: This patient has a spinal epidural abscess causing cauda equina syndrome. Urinary retention with or without overflow incontinence is the most common finding of cauda equina syndrome. Spinal epidural abscesses classically present with back pain, fever, and neurologic deficits. In this case, it is urinary retention. Some risk factors for epidural abscess include injection drug abuse, immunocompromised state, recent spinal surgery, and cancer. This patient has a cardiac murmur and track marks on her arm confirming her history of IV drug abuse. This patient's suprapubic mass is due to urinary retention, not a bladder malignancy. Bladder cancer (A) is most commonly found in older adults with a history of smoking and an occupational exposure to certain chemicals. A necrotic uterine leiomyoma (B) can present with fever and pelvic pain, but is usually the result of a rapidly growing leiomyoma or as a complication of uterine artery embolization. It is not associated with neurologic deficits. A tubo-ovarian abscess (D) is part of the spectrum of pelvic inflammatory infections. Lower abdominal pain and fever are frequently present. In addition, there is unilateral adnexal tenderness.

A 52-year-old man presents to the Emergency Department with complaints of ear pain. Vital signs include T 37.9°C, HR 94, BP 152/80, RR 16. Physical examination reveals the edematous, tender ear. Otoscopy is shown above. Which of the following is true regarding this patient's likely diagnosis? It is uncommon in diabetics and immunocompromised patients Methicillin-resistant staphylococcus aureus is the most commonly identified pathogen Other findings on exam can include trismus and cranial nerve VII dysfunction Treatment requires admission for IV antibiotics

Correct Answer ( C ) Explanation: This patient has necrotizing (malignant) otitis externa, an aggressive infection of the external auditory canal. It typically starts as a simple otitis externa but does not respond to the usual topical antimicrobial therapy. Infection spreads to the deeper tissues of the external auditory canal, involving the cartilage, periosteum, and bone. It is most commonly seen in elderly diabetics and immunocompromised patients. Patients present with severe otalgia, headache, otorrhea, and periauricular edema and pain. Granulation tissue in the floor of the ear canal at the bony cartilaginous junction is characteristic of the disease. Trismus indicates involvement of the masseter muscle or temporomandibular joint. The seventh cranial nerve is usually the first cranial nerve involved and is a serious sign of extension of the infection. Cranial nerves IX, X, XI, can also be involved. The vast majority of cases are caused by Pseudomonas aeruginosa. Blood counts are typically normal. The erythrocyte sedimentation rate (ESR) and C-reactive protein (CRP) are frequently elevated but are not required for diagnosis. CT with contrast or MRI can confirm the diagnosis and evaluate for extension of disease. Necrotizing otitis externa is more commonly seen in diabetics and immunocompromised patients (A). AIDS patients are typically younger, have pathogens other than Pseudomonas and have a worse prognosis. While P. aeruginosa is most common overall, methicillin-resistant Staphylococcus aureus (MRSA) (B) now accounts for about 15% of cases. Ciprofloxacin is the antibiotic of choice. Outpatient management with oral ciprofloxacin, due to its bioavailability and penetration to the bone, is an option for many patients. Hospital admission for IV antibiotics (D) may be necessary for those who are toxic appearing, have cranial neuropathies, or need more aggressive management of their pain or diabetes.

A 43-year-old man presents with altered mental status. His vital signs are HR 113, BP 143/63, T 98.9°F and blood glucose of 750 mg/dl. During your evaluation he has a brief generalized tonic-clonic seizure. Which of the following represents the most important intervention for this patient? Alteplase Insulin bolus and continuous infusion Intravenous normal saline Phenytoin

Correct Answer ( C ) Explanation: This patient presents with signs and symptoms consistent with hyperglycemic hyperosmolar state (HHS) and intravenous fluids should be given aggressively early in management. HHS is a syndrome characterized by dehydration, hyperglycemia, hyperosmolarity and altered mental status. Patients may present with confusion, lethargy, seizures, focal neurologic deficits or frank coma. Pathophysiologically, decreased insulin (or insulin action) leads to gluconeogenesis and increased circulating glucose levels. This in turn draws fluid from the intracellular space into the intravascular space. The resultant osmotic diuresis leads to profound intravascular dehydration, electrolyte abnormalities and hyperosmolarity. Typically, patients will have a blood glucose >600 mg/dl and an osmolarity >350 mOsm/L. Blood urea nitrogen and creatinine are usually elevated. Initial management focuses on supportive care and aggressive fluid resuscitation. Patients with HHS are estimated to be 5-10 liters behind. In addition to fluid administration, electrolyte repletion is paramount. Alteplase (A) is used in the treatment of ischemic stroke. Although HHS can mimic a stroke, alteplase plays no role in management. Phenytoin (D) is a common first line antiepileptic drug but it is contraindicated in HHS because it can impair endogenous insulin release. Unlike in diabetic ketoacidosis, patients with HHS have preserved insulin secretion. While a continuous intravenous insulin infusion and even bolus therapy (B) may be used to help lower the blood glucose level in a controlled manner, intravenous fluids are of greater importance in the initial management of the patient.

A 46-year-old woman with a past history of a DVT was recently diagnosed with Burkitt's lymphoma. Recent blood work revealed a creatinine of 2.3. She is currently hospitalized to receive chemotherapy when she suddenly develops tachycardia to a rate of 130 and oxygen saturation of 91%. Which of the following is the most appropriate test to confirm the diagnosis of pulmonary embolism? Chest X-ray CT angiogram of the chest with intravenous contrast D-dimer V/Q scan

Correct Answer ( D ) Explanation: According to the modified Wells Criteria this patient has a high clinical probability for a pulmonary embolism (PE). This patient has multiple risk factors for a PE including cancer, previous DVT, and immobilization secondary to hospitalization. She also has a heart rate greater than 100 and a decreased oxygen saturation, which are associated with PE. The elevated creatinine is a contraindication to receiving intravenous contrast and therefore a she cannot undergo CT angiography, which is the usual test of choice. Therefore, the most appropriate test for this patient is a V/Q scan. CT angiogram of the chest (B) is the preferred diagnostic modality in patients with suspected PE. However, the test requires that IV contrast is administered, which is contraindicated in patients with elevated creatinine. D-dimer (C) is useful in risk stratification of patients suspected of having a PE. A negative D-dimer rules out PE in patients with low or moderate risk established from Well's scoring. A chest X-ray (A) has poor sensitivity for PE.

A pediatric patient presents to your office for a new patient evaluation. He has a history of Hirschsprung's disease. Which of the following complications is associated with this congenital disease? Diverticulosis Fatty liver disease Fistula Toxic megacolon

Correct Answer ( D ) Explanation: Hirschsprung's disease results from an absence of ganglion cells in the mucosal and muscular layers of the colon caused by the failure of neural crest cells to migrate appropriately. The aganglionic segment of bowel is usually narrowed, with dilation of the proximal normal colon. The disease is four times more common in boys than girls and is associated with Down's syndrome. Toxic megacolon is a complication of Hirschsprung disease. The major clinical manifestations are abdominal pain and bloating, abdominal distension, fever, tachycardia, leukocytosis and abnormal bowel sounds. Abdominal radiography will show significant colon air and dilation. The mainstay of treatment is bowel decompression with NG tube. Additional measures include fluid and electrolyte replacement. Corticosteroids and antibiotics may be necessary. If a patient does not improve within 24 hours, a colectomy with reanastomosis or ileostomy is indicated. Fatty liver disease (B) is a complication of chronic alcohol abuse, not Hirschsprung's disease. Fistula formation (C) is not a direct complication of Hirschsprung's. It is however a complication of any intestinal surgery, abdominal infection or inflammatory bowel disease. Diverticulosis (A) is not a complication of Hirschsprung's. It is associated with a low fiber diet.

Which of the following is a contraindication to the use of metformin in patients with type 2 diabetes mellitus? Age greater than 70 years Insulin use Irritable bowel syndrome Renal insufficiency

Correct Answer ( D ) Explanation: Metformin is a biguanide that reduces the production of hepatic glucose and improves glucose utilization peripherally. For patients newly diagnosed with diabetes mellitus type 2, metformin is the recommended initial therapy along with lifestyle modifications to improve glycemic control. Metformin is contraindicated in patients with elevated serum creatinine. This means a level greater than 1.4 mg/dL in women and 1.5 mg/dL in men. Other contraindications include congestive heart failure, radiographic contrast studies, acidosis and patients who are seriously ill. Age greater than 70 years (A) alone is not a contraindication to metformin use, however normal renal function should be confirmed in elderly patients prior to starting the medication. Metformin may be used as monotherapy or in combination with insulin (B). Side effects of metformin include diarrhea and nausea, but irritable bowel syndrome (C) is not a contraindication to its use.

Which of the following is the first line treatment for scabies? Clotrimazole cream Lindane cream Mebendazole Permethrin cream

Correct Answer ( D ) Explanation: Permethrin cream is the treatment of choice for scabies and is applied from the neck down (include the head when involved) and rinsed off 8 to 14 hours later. Usually this is performed overnight. Scabies is caused by an obligate human parasite. Patients present with a pruritic rash that is often worse in the night. Skin findings include papules, nodules, burrows, and vesicular pustules. The distribution includes the interdigital spaces, wrists, ankles, waist, groin, and axillae. Pruritic nodules around the axillae, umbilicus, or on the penis and scrotum are highly suggestive of scabies. In children the head can also be involved. Look for burrows because these are pathognomonic of scabies and will be the best site to find mites. Scabies is a clinical diagnosis based on the typical rash and history. Lindane (B) was once used as a treatment for scabies but is now banned due to its side effect profile and its increased likelihood to cause seizure. Clotrimazole (A) is an antifungal used for all types of tinea infections of the skin. Mebendazole (C) is used in the treatment of pinworms. None of these 3 options are used in the treatment of scabies.

A 59-year-old woman has furosemide-resistant peripheral edema and ascites. Her medical history is significant for sarcoidosis. You order an echocardiogram which reveals right ventricle with increased wall thickness and decreased cavity size, enlarged atria and a normal appearing left ventricle. Cardiac MRI shows no fatty deposition in the ventricular walls. Which of the following is the most likely diagnosis? Arrhythmogenic right ventricular cardiomyopathy Dilated cardiomyopathy Hypertrophic cardiomyopathy Restrictive cardiomyopathy

Correct Answer ( D ) Explanation: Restrictive cardiomyopathy (RCM) is caused by decreased myometrial compliance in the absence of pericardial disease. Several etiologies exist including infiltrative disorders (e.g. amyloidosis, sarcoidosis and hemochromatosis), autoimmune disorders (e.g. scleroderma and polymyositis), storage disorders (e.g. Gaucher's and Fabry's), toxin exposure (e.g. anthracyclines and radiation), esosinophilic disorders (e.g. Loffler's disease), metastatic cancer and diabetes mellitus. In RCM, myocardial compliance is decreased, ventricular cavity size is decreased and wall thickness is normal or increased in a symmetric fashion. Right-sided heart failure is more common than left sided disease, and as such, patients present more with peripheral edema than pulmonary edema. Common associated findings include tachyarrhythmias, thromboembolism (mural thrombi may be seen on echocardiogram), hepatomegaly and ascites, increased jugular venous pulsation, regurgitant murmurs and S3 or S4. It is also common for patients with RCM to fail diuretic management of peripheral edema. Arrhythmogenic right ventricular cardiomyopathy (A) is an autosomal dominant genetic condition which is characterized by fatty or fibrous myocardial deposition. Manifestations include syncope, arrhythmias and sudden death, and less commonly heart failure symptoms. Dilated cardiomyopathy (B) is characterized by thin walled, large cavity sized ventricles. As compared to hypertrophic cardiomyopathy (C), restrictive cardiomyopathy affects the right ventricle more than the left ventricle. RCM is also more likely to produce biatrial enlargement and a normal left ventricle.

An unimmunized child has had a mild fever for several days. Today, the child is brought to the clinic because of the development of a rash. The rash is a pink discrete macular eruption mostly on the face and trunk. Postauricular and suboccipital lymph nodes are palpable. The child has a low-grade fever, but does not appear sick. Which of the following is the most likely diagnosis? Erythema infectiosum Measles Roseola Rubella

Correct Answer ( D ) Explanation: Rubella (German measles) is a systemic disease caused by a togavirus transmitted by inhalation of infective droplets. Mild symptoms including fever, malaise, and coryza may occur first after exposure. A pink maculopapular rash begins on the head and neck, spreads downward, and fades within 3 days. Posterior cervical and postauricular lymphadenopathy occurs 5-10 days before the rash. The characteristic lymphadenopathy and lack of systemic symptoms of this patient are most consistent with rubella. The presence of lymphadenopathy is not consistent with erythema infectiosum ("fifth disease) (A), which typically presents with fiery red "slapped cheek" appearance, circumoral pallor, and subsequent lacy, maculopapular, evanescent rash on the trunk and limbs. High fever and lethargy are prominent features of measles (B) and help distinguish it from rubella. Measles presents with a prodrome of high fever, coryza, conjunctivitis, irritability, and Koplick spots (small, irregular, and red with whitish center on the mucous membranes). The rash is brick red, irregular, maculopapular beginning on the face and preceding downward, similar to rubella. Continued fever after rash onset and lymphadenopathy are not consistent with roseola (C). Roseola begins with a sudden high fever which subsides in a few days, and just as the child appears to be recovering, a red rash appears. This usually begins on the trunk, spreading to the legs and neck.

A 19-year-old man presents to the clinic with the painless ulcer see in the image above. He reports unprotected intercourse with several partners. What is the recommended first line treatment? Ceftriaxone 250 mg IM x 1 Doxycycline 100 mg PO BID x 7days Levofloxacin 500 mg PO x 7 days Penicillin 2.4 million units IM x1

Correct Answer ( D ) Explanation: Syphilis is caused by the spirochete Treponema pallidum. There are four phases of syphilis: primary, secondary, latent, and tertiary. Primary syphilis is characterized by a painless chancre that begins as a papule and then ulcerates. It is usually solitary and occurs at the site of inoculation. It resolves spontaneously after 2-6 weeks. Patients may also have bilateral, painless, nonfluctuant, inguinal adenopathy. The treatment for primary, secondary, and early (<1year) latent syphilis is benzathine penicillin 2.4 million units IM. Alternative treatments include doxycycline, tetracycline, or erythromycin for penicillin allergic patients. Levofloxacin (C) is not a recommended treatment for syphilis. Ceftriaxone 250 mg IM x1 (A) is the drug of choice in the treatment of gonorrhea (Neisseria gomorrrhoeae). Gonorrhea typical presents with purulent urethral discharge but can be asymptomatic. Doxycycline (B) is an alternative agent for the treatment of syphilis and can be used in those patients who are penicillin allergic.

Which of the following medications can cause hyperkalemia? Furosemide Hydrochlorothiazide Regular insulin Trimethoprim-sulfamethoxazole (TMP-SMX)

Correct Answer ( D ) Explanation: The trimethoprim component of TMP-SMX can cause reversible elevations of both creatinine and potassium. TMP-SMX is a sulfonamide antibiotic that is frequently used in the treatment of a variety of bacterial infections. A decade ago, it was the most commonly prescribed antibiotic for urinary tract infections. However, the increased rate of Escherichia coli resistance to the drug has limited it's use for this indication. TMP-SMX is widely used in outpatient treatment of skin and soft-tissue infections because of its activity against methicillin resistant Staphylococcus aureus (MRSA). Among its many side effects, it can cause elevations in creatinine and hyperkalemia. It should be used cautiously in patients with renal impairment or in pairing it with other drugs that can cause hyperkalemia. Furosemide (A) is a loop diuretic agent that can cause hypokalemia and is often used in the treatment of hyperkalemia with preserved renal function. Hydrochlorothiazide (B) is a diuretic, antihypertensive agent that can lead to hypokalemia. Insulin (C) can cause transient hypokalemia by shifting potassium into cells and it is a mainstay of hyperkalemia treatment.

A 9-year-old girl presents with scalp itching. Physical examination reveals the finding seen in the image above. What treatment is indicated? Clotrimazole Ivermectin Lindane Permethrin

Correct Answer ( D ) Explanation: This patient presents with pediculosis capitis or head lice and should be treated with permethrin. Pediculosis capitis is caused by lice infection. These parasitic lice infest and lay eggs at the base of the hair shaft. Transmission is from person-to-person. It is common in children but uncommon after puberty. Patients will present with intense pruritus, which coincides with the lice feeding and inspection reveals nits (immature lice) firmly attached to the base of hair shafts. The heaviest infection is typically seen behind the ears. Diagnosis is made on clinical grounds or with microscopy. The preferred treatment for pediculosis capitis is with permethrin, which kills the adult louse. Subsequently, the nits must be removed with a vinegar solution and fine-tooth comb. The scalp should be reexamined one week later and repeat therapy at this point may be required. Clotrimazole (A) is an antifungal medication that does not play a role in the treatment of pediculosis. Ivermectin (B) is a second-line therapy for pediculosis capitis and should be reserved for cases in which topical therapy fails. Ivermectin is given as a single dose and repeated 10 days later but it is a second line treatment. In the past, lindane (C) was indicated as first line treatment but now is reserved for treatment failures.

A 32-year-old man presents to clinic with worsening colicky stomach pains, and bloody, mucous streaked diarrhea. He reports he has no fever, nausea or vomiting. His symptoms began vaguely a week ago and have progressed in severity since then. He reports he is otherwise healthy, not on any medications and has not travelled recently. He has no family history of colon cancer and reports the symptoms are so bad he is no longer able to go out in public for fear of having to use the toilet. Stool studies are negative. Colonoscopy reveals edema, friability, mucous and erosions of the mucosal surface of the rectosigmoid region. Which of the following is the most likely diagnosis? Colon cancer Crohn's disease Irritable bowel syndrome Ulcerative colitis

Correct Answer ( D ) Explanation: Ulcerative Colitis (UC) is one of two inflammatory bowel diseases, the other being Crohn's disease. UC is a chronic idiopathic inflammatory bowel disease with recurrent symptoms and significant morbidity. The clinical presentation is highly variable and can mimic other types of disease such as pseudomembranous colitis and other infectious diseases which should be ruled out by stool studies as in the above scenario. The hallmark of UC is bloody diarrhea. Other symptoms include abdominal pain which is often times cramping in nature, tenesmus and fecal urgency. Patients can also present with anemia secondary to bleeding. Ulcerative Colitis is limited to the mucosal lining of the colon and always involves the rectum and spares the proximal gastrointestinal tract. In ulcerative colitis it is characteristic to have periods of symptomatic flare-ups and remissions. The goal of treatment in UC is to get flares into remission as well as provide relief of symptoms. First line treatment for mild to moderate UC is 5-aminosalicylic agents. These agents achieve clinical improvement in 50-70% or patients and remission in up to 30% of patients. Unlike UC, the depth of inflammation in Crohn's disease (B) is transmural as opposed to mucosal. Also in Crohn's the entire gastrointestional tract can be affected as opposed to the limited area affected in UC, which is often, times referred to as left sided colitis. Irritable bowel syndrome (IBS) (C) has symptoms of abdominal pain, cramping or bloating with episodes of diarrhea or constipation for at least 3 days out of the month for the last three months. The course of the disease varies in severity from person to person. IBS is not an inflammatory bowel disease and as such does not affect the colon wall in the same way as ulcerative colitis. Colon cancer (A) is a malignant tumor of the colon that often begins as a polyp. There are no clear signs and symptoms in the early stages of colon cancer which is why screening is recommended. Ulcerative colitis is a risk factor for colon cancer.

A 50-year-old man with a history of hypertension, diabetes and stage III chronic kidney disease with a GFR of 45, presents to the emergency department complaining of shortness of breath and stabbing chest pain for the past hour. Vital signs are BP 145/70, RR 36, HR 115, and pulse oximetry 89% on room air. An ECG reveals sinus tachycardia. Two days ago he returned to Los Angeles on a business trip from China. Which of the following is the most appropriate diagnostic test for this patient? Chest CT angiography Chest radiography Pulmonary angiography Ventilation/perfusion scan

Correct Answer ( D ) Explanation: Ventilation/perfusion scan is the most appropriate diagnostic imaging modality for this patient who likely has a pulmonary embolism. For most patients with suspected pulmonary embolism, chest CT angiography is the first-choice diagnostic imaging modality because it is sensitive and specific for the diagnosis. However, this patient has stage III chronic kidney disease and chest CT angiography is contraindicated. Ventilation/perfusion scanning is reserved for those with suspected pulmonary embolism in whom chest CT angiography is contraindicated such as renal insufficiency with an estimated GFR < 60, contrast allergy, pregnancy or morbid obesity. In general, ventilation/perfusion scanning is a sensitive test for the diagnosis of pulmonary embolism, but is poorly specific due to the high number of false-positive test results. Chest CT angiography (A) is usually the first choice study for diagnosis of a pulmonary embolism, but it is inappropriate in this patient with advanced chronic kidney disease. Certain findings on chest radiography (B) may be suggestive of a pulmonary embolism, but are not considered diagnostic. Pulmonary angiography (C) is the most specific examination available and is considered the gold standard; however it is more invasive and rarely used.


Kaugnay na mga set ng pag-aaral

Unfair Deceptive Abusive Acts or Practices

View Set

Unit 3: [Assessment] Lessons 5, 6, 7 & 9

View Set

Sem. 2 Upper Respiratory condition

View Set

VM 569 Equine Angular and Flexural Limb Deformities (Day 9, part 1)

View Set

Get enterprise features with analytics 360

View Set